Ateista Klub

“Az írástudók és a farizeusok a Mózes székében ülnek: Annakokáért a mit parancsolnak néktek, mindazt megtartsátok és megcselekedjétek; de az ő cselekedeteik szerint ne cselekedjetek. Mert ők mondják, de nem cselekszik.” Máté 23:2-3 "ne figyeljenek oda arra, amit mondok, egyetlen dologra figyeljenek, amit csinálok" Orbán Viktor

Friss hozzászólások

  • Brendel Mátyás: @meglévő alap: és ha a Yule fa a karácsonyfa elődjének számít, akkor sokkal előbb is állítottak. S... (2024.03.03. 22:55) Jézus nem karácsonykor született
  • Brendel Mátyás: @mezgag85: " Izrael biztostja már évtizedek óta Gázának nagyrészt az üzemanyagot, a vizet, az áram... (2024.03.03. 22:33) Eliszlámosodik-e a nyugat még ebben az évszázadban?
  • meglévő alap: A legnagyobb érdekesség számomra az, hogy a Biblia osztogatás legnagyobb ellenzője sokáig a római ... (2024.02.26. 20:42) Éjféli mise
  • Brendel Mátyás: @Mézgagé: nem fogtad fel a postot. a post arórl szól, hogy az Univerzum lehet véges idejű, de mégi... (2024.02.20. 16:58) Univerzum-modellek isten nélkül
  • Brendel Mátyás: @ABCenaturi17: a középkor sötétségét mutatja még a sok kínzás. nemcsak a gonoszság, hanem a butasá... (2023.12.13. 21:58) A legjobb ateista könyv, ever...

Címkék

1 (1) abortusz (2) Ádám és Éva (2) adó (1) agnoszticizmus (9) agresszió (4) AIDS (1) áldozat (4) alkotmány (1) államegyház (6) állatvédők (1) altruizmus (2) áltudomány (3) Amerika (1) analitikus (2) analógia (1) anarchizmus (2) anglia (1) anglikán egyház (1) angyalok (1) animizmus (1) antiszemitizmus (1) antropocentrizmus (2) argumentum ad ignorantiam (1) ateista (10) ateisták (1) ateista egyház (4) ateista párt (1) ateizmus (25) ausztria (1) az ateizmus nem hit (6) a hit ereje (2) a vallások vége (11) a vallás bűnei (2) a vallás vége (8) babona (1) bátorság (2) bayer (1) béke (3) berg (1) bergoglio (3) bertrand russel (1) betegség (9) Biblia (16) biblia (11) bizalom (1) bloggolás (1) boko haram (1) boldog (1) boldogság (10) bolgogság (1) börtön (2) boszorkányüldözés (2) botrány (1) breivik (2) búcsúcédulák (1) buddhizmus (11) bújkáló isten (2) bűnkultusz (2) bűnök (13) bűnözés (37) burka (1) bűvészet (1) cáfolás (1) carl sagan (1) celebek (1) cenzúra (7) cherry picking (1) család (1) csillaghamu (1) csoda (11) csodák (1) dawkins (4) deizmus (7) dekadencia (1) demarkáció (3) demográfia (1) demokrácia (6) Dennett (1) descartes (8) diderot (5) divergencia (16) djihad (2) dogma (1) douglas adams (1) dőzsölés (1) drogok (2) dualizmus (10) dzsihád (2) egészség (1) egyenlőség (2) Egyesült Államok (1) egyház (3) egyházadó (1) egyházállam (20) egyházkritika (5) egyháztörvény (3) egyiptom (1) egzaktság (1) egzisztencializmus (2) eincheitswissenschaft (2) életfilozófia (2) életrajz (2) életszemlélet (5) élet értelme (26) eliminativizmus (1) ellenőrzés (2) ellentmondások (2) elmefilozófia (3) elmélet (1) élmény (3) elnyomás (3) elv (1) empirizmus (7) eq (1) eretnekek (3) erkölcs (29) erkölcsi relativizmus (7) erőszak (11) erotika (2) értékrend (3) értelem (5) értelem és érzelem (8) érvelési hiba (3) érzelem (7) esztétika (3) etika (26) etikaoktatás (2) etikaóra (6) etiopia (1) eu (1) eucharistia (1) evangelium (1) evangéliumok (8) evolúció (13) evolúcó (1) ezotéria (5) facebook (2) fanatizmus (8) fejlődés (1) feltámadás (4) felvilágosodás (6) feminizmus (5) fenyő (1) fetisizmus (1) feymann (1) film (16) filozófia (1) filozófiai racionalizmus (1) filozófus (17) filozófusok (4) finnország (2) finomhangoltság (5) fizika (3) fizikalizmus (4) flow (4) fogalomrendszer (1) függőség (1) fundamentalizmus (6) genezis (1) globalizáció (1) gonosz (21) gy (1) gyerekek (3) gyilkosság (3) gyónás (1) háború (1) hadisz (1) hadith (1) hagyomány (1) halál (19) halál közeli élmény (2) házasság (2) hazugság (1) hedonizmus (1) Heidegger (2) hézagok istene (1) higiénia (1) himnusz (1) hinduizmus (8) hinduk (1) hírcsárda (1) hit (20) hitchens (3) hittan (4) hitvi (1) hitvita (3) hit és tudás (16) hit nélkül élni (8) homeopátia (1) homoszexualitás (8) Hume (2) humor (21) húsvét (1) idealizmus (3) időutazás (1) igazi vallás (1) igazolás (23) igazságosság (2) ikon (1) ima (4) india (5) indonézia (2) indukció (1) inkvizíció (15) instrumentalizmus (2) integráció (1) intellektuális tisztesség (2) intelligencia (4) intelligens tervezés (1) intolerancia (3) irán (1) Irán (1) irodalom (2) irónia (3) irracionalitás (1) isten (11) istenérv (22) Isteni Téveszme (1) istenkáromlás (2) isten halott (1) isten nélkül nincs erkölcs (2) iszlám (46) ízlés (1) izrael (2) játszmaelmélet (1) jézus (23) Jézus (11) jog (1) jóságosság (1) kálvinizmus (2) karácson (1) karácsony (8) karikatúra (1) katasztrófa (1) katolicizmus (11) katolikusok (2) kdnp (3) kereszt (1) keresztelés (1) keresztény (9) keresztényésg (3) kereszténység (38) keresztes hadjáratok (1) kettős mérce (1) Kierkegaard (1) kínzás (6) kivégzés (1) klerikalizmus (2) kognitív disszonancia (1) kölcsey (1) kommunizmus (4) kontinentális filozófia (1) könyv (53) könyvégetés (1) korán (3) koron (1) koronavírus (1) korrupció (1) körülmetélés (3) középkor (6) kozmológia (2) közösség (1) kreacionizmus (7) kreacionmizus (1) kult (1) kultúra (2) legenda (1) lélek (9) lengyelország (1) liberalizmus (3) librivox (1) logika (4) lopás (2) lövöldözés (1) luther (2) magyarázat (2) maher (1) mali (1) mária (2) mártírok (1) maslow (1) matematik (1) matematika (2) materializmus (10) matterhorn (1) mazochizmus (1) medicína (1) meditáció (1) megbocsátás (1) megtermékenyítés (2) mémelmélet (2) menekültkérdés (3) mennyország (12) mérleg (1) mese (3) mesterséges intelligencia (6) metafizik (1) metafizika (11) metafóra (6) metodika (1) militantizmus (1) mise (1) miszt (1) miszticizmus (1) mitológia (2) mítosz (5) modernizáció (2) módsze (1) módszertan (2) monizmus (2) monoteizmus (3) moore (1) mormonizmus (1) mormonok (1) multikulti (3) muszlim (2) mutyi (1) művészet (2) múzeum (1) náci (1) nácizmus (4) naturalizmus (1) NDE (1) németország (4) nepál (1) népek ópiuma (10) népírtás (4) népszámlálás (1) neurobiológia (5) neurózis (2) nevelés (1) nietzsche (4) nobel (1) nők (1) objektív (6) objektív és szubjektív (5) occam (5) okság (1) oktatás (11) ökumenizmus (1) öncsonkítás (1) öngyilkosság (2) önszerveződés (1) ontológia (3) örök élet (2) orvoslás (1) ősrobbanás (1) összehasonlító valláskritika (1) pál (2) palesztína (1) panteizmus (2) pap (1) pápa (4) paradoxon (1) paranoia (3) pascal (3) pedofilia (1) pedofília (3) plágium (1) pogányság (3) pogrom (1) pokol (3) politeizmus (2) politika (19) pornó (2) pozitivizmus (1) predesztináció (1) prostitúció (1) provokáció (2) prüdéria (2) pszichedelikus (1) pszichológia (8) qualia (2) rabszolgaság (2) racionalizmus (7) radikalizmus (1) ratzinger (2) redukcionimzus (2) redukcionizmus (2) reform (1) reformáció (2) regresszió (1) reinkarnáció (3) rejtőzködő isten (2) relativizmus (3) remény (1) reprodukálhatóság (1) repülő (1) Richard Dawkins (2) rossz gyógyszer (2) saeed malekpour (1) sajtószabadság (4) sartre (1) sátán (2) satyagraha (1) sci-fi (2) skizofrénia (1) sorozat (2) spagettiszörny (2) spiritualizmus (4) statisztika (13) Sunday Assembly (3) svájc (1) szabadság (8) szabad akarat (7) szadizmus (3) szaturnália (1) szegénység (1) szekta (2) szekták (4) szekularimzus (4) szekularizmus (40) szemet szemért (1) szent könyv (2) szent tehén (1) szerelem (2) szeretet (6) szex (5) szimuláció (4) szintetikus (2) szintetikus apriori (1) szkepticizmus (2) szólásszabadság (2) szollipszizmus (1) sztoicizmus (1) szub (1) szubjektív (7) szüzesség (1) szűznemzés (2) takonyangolna (1) talmud (1) tanmese (22) tanulás (1) taoizmus (1) társadalom (6) tautológia (1) TED (1) teizmus (1) tekintély (1) tény (1) teodicea (8) teodícea (1) teológia (10) teremtés (2) teremté ember az istent (1) természet (1) természettörvények (3) terroizmus (1) terrorizmus (17) tervezés (1) test és elme (4) tinik (1) tízparancsolat (4) tolerancia (2) történelem (10) történelmi jézus (6) transzcendencia (2) transzcendens (4) tudás (2) tudatosság (2) tudomány (21) tudományfilozófia (34) túlvilág (11) tüntetés (2) tv (1) üdvtörténet (1) újságírás (2) újtestamentum (3) üldözés (5) undefined (2) unitárianizmus (1) Univerzum (5) usa (4) USA (2) utópia (1) üzletegyház (1) vagyon (1) vágyvezérelt gondolkodás (6) vakok országa (3) válás (1) vallás (24) vallásfesztivál (1) vallásháború (7) valláskritika (5) vallások vége (5) vallásszabadság (18) vallástudomány (1) vallásüldözés (4) vallás haszna (5) valószínűségszámítás (3) vámpírok (1) varázslás (2) vasárnap (2) vatikán (9) vatikáni szerződés (5) végítélet (1) végtelen regresszus (3) véletlen (1) véletlen egybeesés (1) vermes géza (1) vicc (2) videó (6) vikingek (1) világvége (1) vita (2) voltaire (1) vulgáris (1) zavargás (2) zene (3) zombi (1) zsid (1) zsidók (11) zuhanó repülőgép (1) Címkefelhő

e-mail: maxval1967@gmail.com

e-mail: popocatepetl@freemail.hu

e-mail: miigyelunk@gmail.com

A szubjektív idealizmus nyomorúsága

Brendel Mátyás 2017.01.02. 06:59

12stone-parsons-blog480.jpg

A szubjektív idealizmust George Berkeley neve fémjelzi, és a legjobb összefoglalójának a "Három dialógust" tartják, amelyben egy bizonyos Philonous és Hylas vitatkozik egymással. A dialógusok az adott korban, és korábban is, Platón óta bevett, viszonylag élvezetes módjai annak, hogy egy filozófus a nézeteit bemutassa. A dialógus előnye az is, hogy a lehetséges ellenvéleményeket jól meg lehet jeleníteni. A dialógus hátránya, hogy nem illik komplex nézetek leírásához. Életidegen lenne, ha az egyik szereplő nagyon hosszan fejtegetne egy nagyon összetett nézetet. Szerintem emiatt hagytak fel később a filozófusok a dialógusokkal. Berkeley Három dialógusában Philonous jeleníti meg a filozófus nézeteit, és Hylas a különféle ellenvéleményeket. A könyv megtalálható a librivoxon hangoskönyv formájában, és mivel régi szöveg, ezért a könyv szövegesen is megtalálható ingyen.

Na most az olvasó mondhatja azt, hogy mit érdekli őt a szubjektív idealizmus? Régi, lejárt lemez, és igazából még a hívők nagy része sem ezt hiszi, hanem inkább dualista. Általában dualista a jelentősebb egyházak hivatalos tanítása is. Aki ma a szubjektív idealizmust hiszi, azt gyakorlatilag elmebetegnek szokták tekinteni. Ritkábban filozófusnak. De mégis érdekes ez a könyv, mert kihívást jelent. Véleményem szerint egy átlagos mai ateista nehezen tudná megfogalmazni azokat az érveket, amelyek egy elkötelezetlen, nagyon logikusan gondolkodó harmadik személyt meggyőznének arról, hogy Berkeley elméletét kell elvetni, és a materializmust/fizikalizmust érdemes elfogadni. Persze az átlagos ateista kapásból tud érveket mondani, de olyat nehezen, ami nem körkörös, azaz nem eleve a saját nézetrendszerén alapul. Például: "hát nézd már, ott van az a kő, ha fejbe váglak vele, fájni fog!". Egy ilyen érv egy szubjektív idealistának vagy egy elkötelezetlen embernek nem elég, mert hát a szubjektív idealista is elismeri, hogy a kő képzete és a fájdalom képzete az ideáink között van.

Bár én ritkán szoktam egy olyan elméletre, amelyet nem tartok elfogadhatónak, mást mondani, mint, hogy szimplán hülyeség, azért el kell ismerni, hogy Berkeley a maga korában, azon a szinten tulajdonképpen egészen zseniális filozófus volt. Amennyiben Berkeley idealizmusát egy olyan provokatív filozófiai hipotézisnek fogjuk fel, amely ellen érvelni kell, akkor Berkeley azért elég érdekes ujjgyakorlat. Bár meg fogom mutatni, hogy miért nem helyes elfogadni az ideológiáját, de arra kiváló példa, hogy a metafizikával milyen messzire el lehet menni.

Érdemes azzal kezdeni, hogy ontológiai szempontból milyen nagy filozófiai elképzeléseket ismerünk. Tulajdonképpen két nagy csoport van: a monizmusok, és a dualizmus. A dualizmus gyakorlatilag egyféle: eszerint van a materialista/fizikai szubsztancia, és van egy lelki szubsztancia, amely esetleg a túlvilággal/szellemekkel/istenekkel is azonos szubsztancia. Ezt a nézetet Descartes nevével szokták fémjelezni. A dualizmus nagy rákfenéje a két szubsztancia kölcsönhatása: amennyiben a két szubsztancia nem két teljesen izolált világ, akkor kölcsön kell, hogy hassanak. Ha pedig kölcsönhatnak, akkor kérdés, hogy egyáltalán ez hogy lehetséges, illetve mit is jelent akkor a kétféle szubsztancia megkülönböztetése? Konkrétan, ha a "lélek" képes fizikai cselekedeteket okozni, akkor mit jelent az, hogy a lélek nem anyagi? Miben különbözik az anyagtól?! De erről már részletesen írtam.

A monizmusokban viszont csak egyféle szubsztancia van. A materializmus arról ismert, hogy ez az egy szubsztancia az anyag, és ennek megfelelően a "lélek" is anyagra redukálható. Ebbe a nézetbe általában nem fér el isten, mert egy anyagi isten elég problémás elképzelés. A másik monizmus a szubjektív idealizmus: eszerint az egyetlen létező szubsztancia a "lélek", és ez vezet Berkeley idealizmusához. Berkeley szerint az emberi lelkek léteznek, és létezik még isten is. Más idealizmusok szerint esetleg csak isten létezik, ez az objektív idealizmus. A szollipszizmus szerint pedig csak egy én létezik, a mindenkori gondolkodó énje. Ezekre itt nem térek ki, mert még Berkeley idealizmusához képest is elszállt elképzelések, és hasonló problémáik vannak.

Az idealizmus és a materializmus abban közös, hogy egyetlen szubsztancia létezik. Az idealizmus előnye a dualizmussal szemben, hogy nem merül fel a mentális okozás problémája: az anyagtalan lélek tud anyagtalan cselekedeteket okozni, hiszen a cselekedetek nem a fizikai világban történnek, hanem csak képzeletben. Így, amikor elhatározom, hogy iszok egy teát, akkor ennek megfelelően azt képzelem, hogy tényleg iszok egy teát, már, amikor ténylegesen "végre is hajtom" az elhatározásom, és nem jön közbe semmi. Hogy hogy jöhet közbe valami? Ez jó kérdés.

Mert egyik oldalról tudjuk, mindennapi tapasztalat, hogy közbejöhet valami. Például nincs tea otthon, nem tudunk vizet forralni, véletlen sót öntünk a teába, és ezért nem isszuk meg, és hasonló dolgok. Egyrészt ezeket tudjuk, tapasztaljuk. De ha minden csak a képzeletünkben játszódik, ha a teaivást csak mi képzeljük el, akkor miért vagyunk olyan hülyék, hogy azt képzeljük, nincs is tea otthon? Miért nem képzelhetjük azt, hogy van? Egyáltalán, miért nem mi vagyunk akkor mindennek az urai? Miért nem képzelünk magunknak vagyont, szépséget, egészséget, élvezeteket, és mindent, ami jót csak lehet?

Hylas ezt a problémát ennyire konkrétan nem veti fel. Én úgy fogalmaznám ezt meg egy kicsit szakszerűbben, hogy a tapasztalataink azt mutatják, hogy van valamilyen realitás, ami tőlünk független. Berkeley - anélkül, hogy ezt ennyire jól részletezné - azt a megoldást adja erre, hogy itt van szükség istenre. Isten a tőlünk független realitás, aki miatt nem kívánságműsor az élet. Mert tulajdonképpen minden képzelgésünket isten vetíti belénk. Ezért nem képzelhetünk csak úgy akármit el, és ezért tűnik úgy, mintha a világ konzisztens lenne, szabályai lennének, és tőlünk független létező lenne. Ez a tőlünk való függetlenség egy külső realitásra utal, de Berkeley nem azt mondja, hogy a tea azért külső realitás, mert létezik, mint fizikai entitás, hanem azért, mert isten elméjében létező entitás, azaz idea.

Philonous és Hylas eljutnak oda a dialógusban, hogy az idealizmus és a materializmus is azt mondja, hogy van a tapasztalataink mögött egy absztrakt valami, ami biztosítja a tapasztalataink konzisztenciáját, és amelynek törvényei vannak, és nem enged meg nekünk szabad képzelgést. Például amikor a teáskannát látom, a sípolását hallom, a kiöntött tea illatát érzem, aztán az ízét is érzem, és annak, hogy mindezek egymással térben és időben korrelálnak, eme tapasztalatok megfelelnek bizonyos fizikai törvényeknek - például a teáskanna akkor sípol, amikor a víz felforr, és gőzzé válik - mindezek Berkeley szerint is azért vannak, mert a tapasztalataink mögött van valami absztrakt entitás (kanna, tea, víz), de ezek nem fizikai tárgyak, hanem isten elméjében lévő entitások. Hylas nagyon sokáig kitart amellett, hogy ezek anyagi tárgyak, Philonus pedig igen nehezen talál jó érvet arra, hogy miért ne lennének azok.

Philonous viszont nagyon sok olyan érvet sorakoztat fel, amelyek csak a naiv empirizmus ellen szólnak. Hylas személyében mindenféle lehetséges ellentétes filozófiai álláspont megjelenik, ezért a dialógus elején ő naiv empirista, aki azt hiszi, hogy például a teásfilter - legyen mondjuk lipton yellow label - sárga színe az ténylegesen a filter egy tulajdonsága. Ezt a naiv elképzelést nagyon könnyen ki lehet kezdeni, és már Locke bevezette erre a problémára az elsődleges és másodlagos tulajdonság megkülönböztetését. A színek valójában nem tényleges tulajdonságai a tárgyaknak, hanem arról van szó, hogy a fény, a tárgy és a szemünk, agyunk viszonyainak megfelelően mi érzékelünk valamiféle színeket, amely színinformáció a tárgyak elsődleges tulajdonságairól közvetett információt szolgáltatnak. Valójában ugye nincsenek sárga atomok, a címke sárgasága a címke fizikai tulajdonságából adódik, de nem közvetlenül. Valójában a címke szerkezetéről, és az abból következő fényvisszaverő tulajdonságairól szól. Locke megkülönböztette az elsődleges tulajdonságokat, például a tárgyak nagyságát, vagy alakját, melyeket tényleg érzékelünk, és nincs különösebben transzformálva. Berkeley ismeri Locke filozófiáját, és ezt is kikezdi. Van abban igazság, hogy még az elsődleges tulajdonságoknál is szerepet játszik a szubjektív faktor, vagy bizonyos relativizmus. Ha egy tárgy nagyságát nézzük, akkor is igaz az, hogy mi egy látszólagos nagyságot látunk, ez függ a tárgy távolságától, függhet a nézőponttól. A tárgy tényleges nagyságára ebben az esetben is visszakövetkeztetünk. Tehát még az elsődleges tulajdonságok érzékelésében is van egy közvetettség, és van egyfajta reláció a személy és a tárgy között. Tehát nem annyiról van szó, hogy ha egy épületnek van egy tulajdonsága, például, hogy 300 méter magas, az közvetlenül belemegy a fejünkbe.

Ugyanakkor, ha mindezeket elismerjük, Berkeley-nak meg el kell ismernie, hogy például, amikor egy tárgyhoz közeledünk, annak látszólagos nagysága törvényszerűen változik, és az a magyarázat, hogy ez amiatt van, hogy a tárgy ténylegesen létezik, és mi a tényleges, fizikai térben közeledünk hozzá, ez egy igen kézenfekvő, és jó magyarázatnak tűnik. Így lesz a tárgyak érzékelésének relativitásából és változékonyságából is egy fizikalista/materialista érv. Berkeley pedig, mint láttuk, végül el kell ismerje, hogy racionális feltételezés az, hogy mindeme szabályosságok mögött van valami absztrakt entitás, de míg a materialista szerint ez egy ténylegesen létező tárgy, addig ő ezt isten elméjébe rakja.

Berkeley előjön pár olyan szofista trükkel is, ami nem érv. Így például sokat lovagol azon, hogy minden tapasztalatunk az elménkben van, hogyan tudnánk kilépni az elménkből egy valós világba, amikor a tapasztalataink, gondolataink, minden az elménkben van? Berkeley itt nem tudom, mennyire komoly, de egész egyszerűen nem képes felfogni azt, hogy a gondolataink, fogalmaink referálni tudnak a külső világra. Tehát, miközben a teáskanna fogalma a fejünkben van, addig egy valós teáskannára referál (utal), és annak bizonyos mértékben meg kell, hogy feleljen ahhoz, hogy azt mondhassunk, a tudásunk igaz (ez az igazság korrespondencia elmélete). Berkeley pedig mintha ezt nem tudná felfogni. Nem mintha Berkeley idejében a korrespondencia elmélet ilyen szakszerűen ki lett volna dolgozva, de egy hétköznapi szinten akkor is fel lehetett ezt vázolni, és nem kellett volna, hogy Berkeley-nek gondot okozzon. Nem csinálhatott volna ebből rossz érvet.

Hogy ez az érv miért rossz, és ezzel már Berkeley rendszerének durva hibáit is felhozom, az az, hogy Berkeley szerint léteznek elmék. Léteznek tehát különféle emberek. És amikor én azt mondom, hogy "George Berkeley, a híres filozófus", akkor az Berkeley szerint is egy olyan elgondolás, amely referál valamire, ami az én elmémen kívül van, méghozzá rá. Ha pedig én képes vagyok az elmémen kívül létező George Berkeley-ról beszélni, rá referálni, sőt, róla megtudni dolgokat, akkor ez a teáskanna esetében sem lehet fogalmilag problémás. Ezt az érvet Hylas nem veti fel, de ha felvetette volna, akkor Berkeley kénytelen lett volna elismerni, hogy a gondolataink referálhatnak az elmén kívüli dolgokra, mert ha más nem, akkor más személyek az elménken kívül vannak.

Berkeley kénytelen elismerni, hogy nem lehetséges olyan érve, ami logikailag cáfolná a materializmus lehetségességét. Egy mai materialista, a mai tudásunkkal felfegyverkezve Berkeley minden észrevételét meg tudja magyarázni. Az érzékcsalódásoktól kezdve a másodlagos tulajdonságokig. Sőt, a materializmusból következik az, hogy ha a tárgyak rajtunk kívül léteznek, és mi rendelkezünk érzékelési képességekkel, akkor az az érzékelés közvetett kell, hogy legyen, és mindazok az érzékcsalódások, illetve az érzékelés relatív, szubjektív volta szükségszerű.

Cserébe mi is elfogadhatjuk, hogy nem létezhet olyan érv, amely egy az egyben cáfolná Berkeley idealizmusát. Azért nem, mert a materializmus és az idealizmus metafizikai elképzelések, és empirikusan ekvivalensek. Philonous  és Hylas a dialógusban nagyjából eljut ide, amikor belátják, hogy az, hogy az absztrakt entitásokat tárgyaknak nevezzük, vagy isten elméjében lévő ideáknak, az első nekifutásra csak egy elnevezés kérdése. Modern szóhasználattal élve, empirikusan ez nem ellenőrizhető. A logikai pozitivisták álláspontja, miszerint az ilyen viták értelmetlenek, ezen a felismerésen alapszik.

Minden olyan érv, amely a materializmus vagy az idealizmus mellett szól, legfeljebb olyan jellegű lehet, hogy melyik elképzelés praktikusabb, melyik egyszerűbb, melyik az, amelyikkel jobban tudunk gondolkodni. Berkeley úgy látszik, határozottan azt gondolja, hogy a materializmus antiintuitív, azaz neki nehéz elképzelnie, és, hogy a szubjektív idealizmusa az egyszerűbb. Ugyanakkor elfogadja, és Hylas fel is emlegeti, hogy az emberek ösztönösen fizikalisták. Gyakorlatilag minden ember úgy beszél a mindennapokban, miszerint teáskannák tényleg vannak. És nehéz úgy beszélni, hogy "az isten elméjében lévő teáskannáról most úgy képzelem, hogy öntök egy kis teát, és isten ezt megengedi, ezért úgy fogom képzelni, hogy öntöttem magamnak egy kis teát". Majdnem minden ember automatikusan azt mondja, hogy: "öntök a teáskannából teát", és ezeket valós, fizikai tárgyaknak tekinti. Az emberi agy az evolúció során így alakult ki, ezért automatikusan így gondolkodik. Ez pedig azt igazolja, hogy a fizikalista elképzelés az evolúció során jól működött, ezért van bennünk valószínűleg genetikai mélységekig kódolva.

Szerintem egyértelműen Berkeley idealizmusa az, ami kacifántos, és szükségtelenül bonyolult. Egyrészt van ez a furcsa beszédmód. Aztán nézzük meg a komplexitást! A materializmus szerint van az Univerzum, és abban az emberek, és ennyi. Berkeley szerint vannak az emberek, van egy isten (ez eggyel több, szükségtelen dolog), és az egész Univerzum benne van isten elméjében, és isten leszimulálja nekünk pont úgy, mintha valóság lenne. Tehát virtuális valóságként létezik az Univerzum, a teljes komplexitásával. Egyértelmű, hogy Berkeley elképzelése a bonyolultabb, eggyel több létezőt hipotetizál: istent.

Aztán meg Berkeley világában mi is ez az isten? Mik az emberi elmék, hogy működik a kapcsolat isten és az emberi elmék között? A mi Univerzum-szimulációnk csak az elmékben vannak, de miben vannak az elmék?

Aztán vannak furcsaságok, és nehezen megmagyarázható dolgok: például - és a dialógusban egy ilyen téma is felvetül, de nincs kibontva - isten valamiért úgy szimulálja nekünk ezt az univerzumot, hogy nemcsak a teáskanna létezik, de létezik a fény, létezik a szemünk, ahova a fény bejut, az agyunk, amely feldolgozza a jeleket. Miközben Berkeley szerint az agyunk, fizikai tárgyként nem létezik, és miközben az agy a materialista hipotézis egy igen fontos része, eközben isten ezt leszimulálja. A komplett Univerzumot olyan mértékben materialistának szimulálja, hogy még azokat a részleteket is leszimulálja, amik kellenek ahhoz, hogy egy materialista Univerzumban meg tudjuk magyarázni, hogy a materializmus hogy igaz. Istennek nem kellene agyakat szimulálnia. Minek?! Az agy nem valami külső dolog, amit élvezünk, amivel játszunk, hanem a gondolkodás szerve, ami csak a materialista elképzeléshez kell. Isten mintha tökéletesen megtévesztő szimulációt csinálna. Ez pedig így egy üldözési mániás, beteges hipotézis. Ez pont olyan, mint amikor a paranoiás például azt képzeli, hogy a buta tehénfejő parasztlány azért viselkedik buta, tehénfejő parasztlányként, mert egy CIA ügynök, aki olyan rafinált, hogy buta, tehénfejő parasztlányt szimulál.

És ezekkel a meggondolásokkal szerintem egyértelmű, hogy miközben persze a szubjektív idealizmus egy logikailag elképzelhető metafizika, aközben egy feleslegesen bonyolult összeesküvés-elmélet. Ilyenből pedig Dunát lehetne rekeszteni, és hogy rendet teremtsünk, azért fogadtuk el az Occam borotvájaként ismert elvet, vagy szakszerűbben mondva, a tudományfilozófia ökonomikussági elveit, és ezek által egyértelműen nem Berkeley elmélete az, amit el kell fogadnunk, hanem a fizikalizmus/materializmus.

Berkeley a katolikus Írországban élt, bár ekkor épp brit, azaz anglikán megszállás alatt. A könyv ajánlásában az az érdekes, hogy Berkeley mennyire benyal Lord Berkeley-nak of Strattonnak. És érzékelhető az is, hogy tartott attól,hogy nézeteit eretneknek minősítik. Berkeley valószínűleg őszintén hitt istenben, de például az a rész, amikor előadja az azóta már szokásos szöveget arról, hogy a természet mennyire harmonikus, és ez istenre utal, az ma már elég szirupos baromság.

Van még egy hely, ahol Berkeley vallásos meggyőződésből olyat csinál, ami méltatlan a filozófiájához, amely legalább egy méltó kihívás volt. Hasonlóan méltatlan dolog az, ahogy a Genezist próbálja összeegyeztetni az elméletével. A Genezis problematikáját láthatóan nekiszegezte már Berkeley-nak, és választ akart adni. A probléma ugye az, hogy a Genezisre Berkeley nem mondhatja, hogy hamis, téves, mert ez eretnekség lett volna. Márpedig a Genezis tárgyak teremtéséről szól. Philonous első megközelítése, hogy a Genezis nem tárgyak teremtéséről szól, hanem a Hold, a Nap teremtéséről, amelyek ideaként tényleg léteznek isten elméjében, és az emberek elméjében. Itt látszólag csak szavakon való lovaglásról van szó, de az igazi probléma, hogy hogy lehetne az ilyen dolgok teremtését értelmezni? Ha a teremtett dolgok isten elméjében is vannak, akkor azoknak mindig is kellett ott létezniük, mivel minden változás istenben tökéletlenség is volna egyben. Így tehát isten elméjében mindig is léteztek a dolgok, ez nem lehet a teremtés jelentése. Egy lehetséges válasz volna, hogy a Hold teremtése azt jelenti, hogy a Holdat isten megjeleníti az emberek elméjében. Csak hát az a bibi, hogy a Genezis tök egyértelműen azt mondja, hogy a Holdat isten az emberek előtt teremtette. Philonous megoldása, hogy isten az ember előtt más véges lények elméjében jelentette meg a dolgokat. Ez már eleve egy erőltetett megoldás, mivel a Biblia ezt így nem írja. De van itt egy nagyobb probléma is. Hylas nem veti ellen, de itt megint arról van szó, hogy mik is akkor az emberi elmék? Az emberi elmék nem lehetnek csak képzelt dolgok, mert az elmén kívül léteznek. Akkor mit jelent az emberi elmék létezése? Azt, hogy más elmékben jelentek meg? Csakhogy ez csak egy regresszió, amely szükségképpen feloldhatatlan az első teremtett elménél: az nem jelenhetett meg másik elmében. Igazából Berkeley ontológiája itt alapvetően tarthatatlan, az emberi elmék "a levegőben lógnak".

Philonus tovább szövi a dolgot, ez visszakapcsolódik az előző problémához, csak ez általános. Philonus elismeri, hogy az problémás, hogy istennek örök idők óta az elméjében voltak elképzelések, aztán egyszer csak megteremtette őket. Ez egy változás, és ez ellent mond isten tökéletességével. Philonus megjegyzi, jogosan, hogy ez nem is az ő elméletének a baja, hanem a teremtés, mint olyan problémája. Valóban egy dualista elképzelésnél is ugyanilyen problémás. Philonus "megoldása" az, hogy van itt egy ellentmondás, de isten végül is felfoghatatlan.

Na most ez az a pont, amikor azt mondhatjuk, hogy Berkeley sokkal egyszerűbben elintézhette volna a dolgot azzal, hogy a Genezis téves. De ez eretnekség lett volna. Berkeley ettől félt. A második problémánál is azzal védte le magát, hogy aki azt meri állítani hogy a teremtés és a tökéletes isten képtelenség, az maga az eretnek. Ma már nem lehet megmondani, hogy Berkeley itt csak az eretnekség ellen védte magát, miközben valószínűleg már felismerte, hogy a Biblia legalább részben hülyeség, vagy titokban esetleg már nem is hitt istenben, de azt tudta, hogy egy olyan könyvet már nem adhat el, ami ide vezet.

Címkék: könyv idealizmus ontológia occam

> komment

A bejegyzés trackback címe:

https://ateistaklub.blog.hu/api/trackback/id/tr4812055857

Kommentek:

A hozzászólások a vonatkozó jogszabályok  értelmében felhasználói tartalomnak minősülnek, értük a szolgáltatás technikai  üzemeltetője semmilyen felelősséget nem vállal, azokat nem ellenőrzi. Kifogás esetén forduljon a blog szerkesztőjéhez. Részletek a  Felhasználási feltételekben és az adatvédelmi tájékoztatóban.

Koós István 2017.01.02. 09:23:44

Nem tudom, miért, de az én képernyőmön nem látszik a szöveg, az egész szürke, meg kicsi is.

Brendel Mátyás · http://ateistaklub.blog.hu/ 2017.01.02. 10:21:35

@Koós István: ja, kösz, véletlenül valahonnan belekerült egy ilyen színformázás, és a szerkesztőben nem látszik

Untermensch4 2017.01.09. 00:08:40

"Igazából Berkeley ontológiája itt alapvetően tarthatatlan, az emberi elmék "a levegőben lógnak"."
Akkor nem (és ez az emberek előtt dolgok teremtését/leképezését is magyarázhatná) ha kezdetben tényleg csak isten volt de tök egyedül és semmi más. A nagy semmiben (ami nincs is tehát nem benne de csak így tudom mondani)egyetlen tudat, nyilván "tökéletes", nem nehéz egyetlen indulóként a verseny legjobbjának lenni. Egy teljesen külső kontroll nélküli elme (mivel semmi nincs amihez képest minősíthető) "tökéletes" (mivel idő sincs, hiszen saját magán kívül időként érzékelhető változás sincs). Amíg az említett külső kontroll-hiány miatt nem zakkan meg és kezd fantáziálni teljesen felesleges dolgokat, pl egy univerzumot. De mivel rajta (és a fantáziáján kívül) nincs semmi, a megőrülés folyamatában is "tökéletes". "Lehasad" róla egy másik személyiség, de csak képletesen, az elméjében, jól elbeszélget vele. Lehasadhat több is. Tképp hozzá hasonlóak... szabad akaratuk van, rá kell vennie őket hogy menjenek szépen vissza, de hiába tökéletes ő maga, ha ő "teremtette" a leszakadt személyiségeket és azok hasonlóak hozzá akkor hiába akarná "izomból" eltüntetni őket, nem szuicid ő szegény, csak kicsit megzakkant.
Bocs hogy ilyen zavaros, és tudom hogy hülyeség mert nem igazolható vagy cáfolható . De egy ilyen "csak isten létezik de megőrült és mi vagyunk a tünetei", eretnekségnek nagyon ízletes lenne, nemde?

Untermensch4 2017.01.09. 21:03:39

@Brendel Mátyás: Megpróbálom újra... :) Occam borotvája alapján a "legegyszerűbbnek" az tűnik hogy egyetlen dolog legyen és semmi más, isten. Aki megőrült és minden más csak ennek "tünete", a fantáziájában. Nem vizsgálható de tényleg bármilyen "észlelésünket" meg lehet vele magyarázni. Ha legközelebb letámad egy hittérítő, remélem eszembe jut lesokkolni ezzel. :)

Brendel Mátyás · http://ateistaklub.blog.hu/ 2017.01.10. 07:55:00

@Untermensch4: ha az Occam borotvája elvet nagyon primitíven veszed, akkor nyilván az egyetlen dolog létezik, az isten, az a minimum, mert egy létező van. de még így is ki tudom kezdeni a dolgot, mert ezzel teljesen egyenértékű az, hogy: "egyetlen dolog létezik, az Univerzum".

valójában ugye nem egyértelmű az, hogy amikor sok létezőt belecsomagolsz egy létezőbe, akkor jogos-e azt mondani, hogy egy létező van.

és ha egy körültekintőbb mércét veszel, akkor ugye nem azt kell nézni, hogy hány léteződ van, hanem inkább azt, hogy a modelled mennyire komplex, mennyire magyaráz meg dolgokat, és mennyire tud predikálni, és milyen pontosan.

na most az őrült istened elméjében ott vannak az emberek, és ott van még a teljes Univerzum. a komplexitása tehát továbbra is eggyel nagyobb, mint a materialista modellnek, mégpedig azzal, hogy nálad ott van isten, a materialista modellben meg nincs isten. az emberek és az Univerzum komplexitása nálad is ott van, csak bele van csomagolva istenbe, de ettől a komplexitás nem fog csökkenni.

a te elméleted magyarázóképessége valóban korlátok nélküli, hiszen az istened őrült, bármi kitelik tőle, de ahogy Popper kifejtette, egy elmélet nem attól jó, ha bármit meg tud magyarázni, hanem attól, hogy mennyi mindent tud kizárni. nem csak azt kell tudni magyarázni, hogy miért az van, ami, hanem még jobb azt megmagyarázni, hogy miért nincs más. illetve ami ugyanez, az elmélet predikciós képessége is számít: mennyire tudod jól megjósolni, hogy mi lesz? egy őrült istennél semennyire. a materialista, és a tudományos elméletek által leírt Univerzum esetében a tudományos tudásunk kidolgozottságához mérten elég jól tudunk ilyeneket jósolni, illetve meg tudjuk sokszor magyarázni, hogy miért nem történt valami más.

szóval az elméletek modernebb mércéjével mérve a te elméleted elég szar. nagyjából a "bármi lehet, mert csak" szintjén van.

Untermensch4 2017.01.10. 09:56:09

@Brendel Mátyás: Igen, eléggé szar, sőt nagyon. Nem tudtam volna ilyen frappánsan összefoglalni mint te, hogy 1db istennel komplexebb mint kéne. Nem is filozófiai hanem patológiai érdekessége miatt miatt látok benne lehetőséget, erre a speciális területre, a hittérítők zaklatására. A racionális érvekre a valláskárosultságuk miatt "immunisak", a saját gondolatmenetük alapján sztem jobban lehet rájuk hatni.
Mint amikor legjobb barátomhoz becsengettek jehovatanúi, neki meg volt egy negyed órája a kísérletre. Szépen elbeszélgettek, olyan kérdéseket tett fel ami alapján úgy tűnt hogy érdeklődő, megtéríthető. Aztán hogy így elaltatta a gyanakvásukat jött a záró keresztkérdés: szóval 144ezren mennek a mennyek országába amikor jön a végítélet... ti jehova tanúi meg világszerte vagytok 10-20 millióan..? :)
Ilyen helyzetben az érvelési hibák nem annyira számítanak mint hogy ne "külső" hanem saját dogmarendszerük "belső" információival kínálja meg őket az ember. :)

Atomic Warning 2017.01.10. 10:53:59

@Brendel Mátyás: A hívőknek és az ezósoknak is van egy téves alapállása, amivel annak ellenére is nehéz vitázni hogy nyilvánvalóan téves. Ezek a: "minden mindennel összefügg" "minden rezgés" "semmi sem lehetetlen". Ezekkel nyitnak vagy csak simán válaszként használják ezeket és az ilyen értelmű szófordulatokat úgy, hogy cáfolhatatlan tényként kezelik ezeket, és globális érvényességűnek. Nehéz ellene érvelni bár nyilván ökörség mindegyik, viszont érvelés nélkül ezekre felépítik a komplett téveszméjüket. Pedig ezeknek az érveknek a rövid cáfolata az egész érvrendszerüket összedönthetné.

brandon1 2017.05.04. 13:09:32

"a teáskanna fogalma a fejünkben van, addig egy valós teáskannára referál (utal), és annak bizonyos mértékben meg kell, hogy feleljen ahhoz, hogy azt mondhassunk, a tudásunk igaz" Ez a gondolatmenet valójában nem a szubjektív idealizmus, hanem a materializmus nyomorúságára mutat rá. A teáskanna fogalma a materialisták szerint egy fizikai jelenség az agyban (pl. egy bizonyos agyterület aktivitása). Egy fizikai létező önmagában véve nem képes referálni semmilyen tőle különböző fizikai létezőre sem. Ahhoz, hogy elmondhassuk, hogy referál valami másra, szükség lenne egy értelmezőre, aki interpretálja az adott agyi állapotot, mint olyasmit, ami a teáskannára referál. De ha ez az értelmező szintén egy agyterület (agyi jelenség), ahogy azt a materialistának szükségképpen feltételeznie kell, akkor végtelen regresszus áll elő, hiszen ennek az értelmezőnek vonatkoznia (referálnia) kell a világban lévő teáskannára mint fizikai tárgyra és a fejben levő fizikai jelenségre is, ahhoz, hogy elmondhassuk, hogy az ő számára az előbbi az utóbbira referál.

"Berkeley kénytelen elismerni, hogy nem lehetséges olyan érve, ami logikailag cáfolná a materializmus lehetségességét" Nocsak, miért lenne kénytelen. Van ilyen érve:

en.wikipedia.org/wiki/A_Treatise_Concerning_the_Principles_of_Human_Knowledge:

"It is impossible to understand what is meant by the words absolute existence of sensible objects in themselves. To speak of perceived objects that are not perceived is to use words that have no meaning or to utter a contradiction.

"Material substance" has two meanings: "being in general" and "support of accidents." (The word accident is used here to mean an unessential quality.) "Being in general" is incomprehensible because it is extremely abstract. To speak of supporting accidents such as extension, figure, and motion is to speak of being a substance, substratum, or support in an unusual, figurative, senseless manner. Sensible qualities, such as extension, figure, or motion, do not have an existence outside of a mind.

Brendel Mátyás · http://ateistaklub.blog.hu/ 2017.05.04. 17:06:11

@brandon1: "Egy fizikai létező önmagában véve nem képes referálni semmilyen tőle különböző fizikai létezőre sem."

de.

" Ahhoz, hogy elmondhassuk, hogy referál valami másra, szükség lenne egy értelmezőre, aki interpretálja az adott agyi állapotot, mint olyasmit, ami a teáskannára referál."

Nem. Ahhoz, hogy szó szerint elmondhassuk, azaz ténylegesen mondjuk a szánkkal, hogy valami referál valamire, nyilván léteznünk kell, és értelmeznünk kell a dolgot. De a dolog akkor is referál valamire, ha mi nem értelmezzük úgy, hogy referál, és nem mondjuk ki a szánkkal.

Az "ahhoz, hogy elmondhassuk" szónak persze van egy másik, elvont jelentése, ami tényleges beszélőt nem feltételez, de és ez azt jelenti, hogy "ahhoz, hogy igaz legyen". De mivel itt nincs szó tényleges beszélőre, ezért ilyen értelemben értve a dolgot, az "érvednek" nincs is alapja, mert ez esetben pont nem szükséges az, amit szükségesnek mondasz. Ebben az esetben nem kell beszélni a dologról, ezért nem kell egy értelmező második személynek lennie, aki beszél a dologról, elég az első személy agyában lejátszódó referálás.

Egy agyterület aktivitása például akkor referál valamire, ha azzal a valamivel igen nagy korrelációban van. Azaz, ha egy agyterület akkor és csak akkor aktív, amikor a nagymamát látom, akkor az az agyterület a nagymamára referál. Vagy egy aktív példa: ha egy agyterület aktivitását mindig egy pofonütés kíséri, és az agyterület csak ilyenkor aktív, akkor az az agyterület a pofonütésre referál.

Te összekevered a referálást, és a referálásról való tudásunkat. A referálásról való tudásunk ugyanúgy tudás, mint minden más tudásunk, nyilvánvaló tautológia, hogy a tudásunk tényleg nem lehet jelen anélkül, hogy mi magunk lennénk, akinek a tudásáról van szó. De a tudás tárgya jelen van akkor is, ha mi nem tudunk róla.

A te hibád az, hogy azt hiszed, a referálás nincs meg, ha senki nem értelmezi. Ami kb olyan hülyeség, mint azt mondani, hogy ha senki nem hallja, akkor az erdőben eldőlő fa nem is recseg.

Persze a szubjektív idealizmusban ez lehet, hogy agy van, de akkor te az "érvelésedben" már eleve feltételezted a szubjektív idealizmust, azaz az "érvelésed" körkörös.

"It is impossible to understand what is meant by the words absolute existence of sensible objects in themselves. To speak of perceived objects that are not perceived is to use words that have no meaning or to utter a contradiction."

Ez megint a "ha senki nem hallja, akkor az erdőben eldől fa nem recseg" típusú baromság. Aki nem szubjektív idealista, annak ez az "érv" eleve olyan égbekiáltó baromság, hogy hihetetlen.

""Material substance" has two meanings: "being in general" and "support of accidents." (The word accident is used here to mean an unessential quality.) "Being in general" is incomprehensible because it is extremely abstract. To speak of supporting accidents such as extension, figure, and motion is to speak of being a substance, substratum, or support in an unusual, figurative, senseless manner. Sensible qualities, such as extension, figure, or motion, do not have an existence outside of a mind."

Itt is a szó, és a referenciájának bugyuta összekeveredéséről van szó. Természetesen a "mozgás" fogalma az "agyunkban" van, de a mozgó test nem az agyunkban van.

Továbbra is csak azt mutatod, hogy milyen ostoba vagy.

brandon1 2017.05.04. 20:03:51

@Brendel Mátyás: "Egy agyterület aktivitása például akkor referál valamire, ha azzal a valamivel igen nagy korrelációban van.

A füst a tűzzel igen nagy korrelációban van, mégse referál rá.

"Azaz, ha egy agyterület akkor és csak akkor aktív, amikor a nagymamát látom, akkor az az agyterület a nagymamára referál."

Ebben a mondatban már mást állítasz, nem az igen nagyfokú korrelációról hanem szigorú állandó együttjárásról írsz. Azt ugyan még senki nem igazolta, hogy lenne ilyen agyterület, de tegyük fel mégis, hogy van ilyen. Még ekkor se igaz az állításod, hiszen még a tökéletes korreláció se referálás. A másik, hogy ha az agyterület aktivitását a nagymama látványa okozza, akkor az egyrészt nem korreláció, hanem ok-okozati kapcsolat, másrészt az agyterület a látókéregben van, ezért nem lehet azonos a fogalommal. A fogalmat ugyanis olyankor is használni kell tudni, amikor nem látjuk a nagymamát, például, vaksötétben is gondolhatunk rá, ezért a használatának nem lehet feltétele az, hogy látjuk őt. Ezért ezzel egyáltalán nem magyaráztad meg, hogyan referálnak a fogalmaink.

"ha egy agyterület aktivitását mindig egy pofonütés kíséri, és az agyterület csak ilyenkor aktív, akkor az az agyterület a pofonütésre referál." Egy frászt referál rá, úgy látszik te nem tudod, mi a referálás. Ez egyrészt nem szükséges feltétel, mert pl. a nemlétező dolgok, mint boszorkány és tündér fogalmai különböznek egymástól, annak ellenére, hogy soha nem kíséri őket a valóságban semmi. Másrészt nem is elégséges, mert ilyen alapon azt kellene mondanunk, hogy Pavlov kutyájának nemcsak az agyterülete, hanem a nyáladzása is az ételre referál ami abszurd. Hiszen, nincs releváns szempontból különbség az agyterület aktivitása és a nyáladzás között, mindkettőt az ételre való kondicionálás váltja ki.

"De a dolog akkor is referál valamire, ha mi nem értelmezzük úgy, hogy referál"

Ez is abszurd. A referálás az nem olyasmi, amit egy fizikai dolog csak úgy csinál magától. Ahhoz minimum asszociálódnia kell valamivel egy elmében, de még ez sem elég hozzá.

"De a tudás tárgya jelen van akkor is, ha mi nem tudunk róla." A referálásnál nincs jelen, mert nem létezik olyan, hogy értelmező nélküli referálás. Az ismerős példa lehet, hogy ha miután kipusztul az emberiség és a szél egy szó betűit fújja a homokba, az nem fog jelenteni már semmit, mert akkor az már csak egy egy természeti képződmény. Miért nem referál még akkor is arra, mint most, hacsak nem azért, mert akkorra már nincs ember, aki értelmezze?

"Ami kb olyan hülyeség, mint azt mondani, hogy ha senki nem hallja, akkor az erdőben eldőlő fa nem is recseg."

Ez mitől hülyeség? Recsegés=hangérzet. Ha nincs megfigyelő, nincs hangérzet sem.

szemet 2017.05.04. 21:14:59

@brandon1: Gondolkozz nem a szubjektív nézőpontból. Mikor gondolod hogy valaki referál valamire? Akkor ha feltételezed, hogy van egy modelje ami megfigyelhető viselkedését úgy befolyásolja mintha referenciát tartana fenn bizonyos dolgokra (amikre te is fenntartasz). Máshogy nem tudhatod van-e bárkinek rajtad kívűl bármire referenciája.

Konkrét példa:

Azaz a kutya nyála nem referencia, mert nem ismerünk mechanizmust ahogy a nyál a viselkedést befolyásolja. Szemben az idegi mintázatokkal amik meg referenciák. De ugyanígy egy önvezető autó nyilván referál a programjában egy addot gyalogosra különben hogyan fékezhetne le előtte???

szemet 2017.05.04. 22:09:55

@brandon1: Ja és a kérdés, ha az önvezető autó referál (adott esetben a mérnök meg is tudja mutatni hol kell változnia a programnak ha gyalogos van az úton, és hogy alakul ki a reakció) akkor van dualista lelke? És ha igen hogyan/mikor/miért kap egy állapotgép lelket?

Brendel Mátyás · http://ateistaklub.blog.hu/ 2017.05.05. 00:49:16

@brandon1: "A füst a tűzzel igen nagy korrelációban van, mégse referál rá."

Én nem azt mondtam, hogy bármi referálhat bármire, ami korrelál vele. A korreláció másnak is lehet a jele. De agy esetében referálást jelent, a tűz és a füst esetében okozást jelent. A tűz nyilván nem referál, mert nem egy komplex információs rendszer. Ezt azért egy 90-es IQ-val fel lehet fogni, és akkor nem kérdezel hülyeségeket.

"Ebben a mondatban már mást állítasz, nem az igen nagyfokú korrelációról hanem szigorú állandó együttjárásról írsz."

Nem ez a fontos kérdés. Itt pontatlan voltam, a korreláció a helyes megfogalmazás. Nyilván lehet mérési hiba az agyaktivitás mérésében, illetve lehetnek hibák az agyműködésben. Szóval csak korrelációt tehetünk feltételnek. Itt a két irányt akartam hangsúlyozni a korrelációban.

" Azt ugyan még senki nem igazolta, hogy lenne ilyen agyterület"

De. Nagyon sok neurális korrelátumot igazoltak. Attól, hogy egy tájékozatlan hülye vagy, még nem lesz igazad.

en.wikipedia.org/wiki/Neural_correlate

"Még ekkor se igaz az állításod, hiszen még a tökéletes korreláció se referálás."

Most g hasból írtál valamit, ami nem igaz, hülyeség, és semmi nem támasztja alá. Amit csinálsz, az nem egy érdemi vita.

" A másik, hogy ha az agyterület aktivitását a nagymama látványa okozza, akkor az egyrészt nem korreláció, hanem ok-okozati kapcsolat"

Az ok-okozati kapcsolatot a korrelációk alapján ismerjük fel. Amikor korrelációt látunk, akkor ennek magyarázatára ok-okozati elméleteket dolgozunk ki. Az ok-okozat iránya az persze kérdés, de nem szükséges ebbe bele menni. A füst és a tűz esetében is erről van szó, de nem mondjuk, hogy a tűz reprezentálja a füstöt, mivel a tűz komplexitása nem olyan, hogy ezt feltételezzük.

"másrészt az agyterület a látókéregben van, ezért nem lehet azonos a fogalommal."

Valójában a "nagymamát" felismerő neurális korrelátum nem a látókéregben van, annál fentebb. Amennyiben valaminek a neurális korrelátuma a látókéregben van, akkor az egy vizuális fogalom, tehát igen, ott van reprezentálva.

" A fogalmat ugyanis olyankor is használni kell tudni, amikor nem látjuk a nagymamát, például, vaksötétben is gondolhatunk rá, ezért a használatának nem lehet feltétele az, hogy látjuk őt. Ezért ezzel egyáltalán nem magyaráztad meg, hogyan referálnak a fogalmaink."

Az, hogy a képzeletbeli képek hogyan alakulnak ki az agyban, egy bonyolult, és még nem eldöntött kérdés.

en.wikipedia.org/wiki/Mental_image#Formation_in_the_brain

De még ha bonyolultabb is, és esetleg kettős kódolás van is, az nem cáfolja a referálást, csupán azt jelenti, hogy kettős referálás van.

"Ez egyrészt nem szükséges feltétel, mert pl. a nemlétező dolgok, mint boszorkány és tündér fogalmai különböznek egymástól, annak ellenére, hogy soha nem kíséri őket a valóságban semmi."

Amennyiben valaki nem hisz a boszorkányokban, akkor a "boszorkány" fogalma konkrétan nem is referál semmire. Amennyiben valaki éppen boszorkányt éget el a máglyán, mint azok a jó kis középkori keresztények, akkor a "boszorkány" fogalma arra a nőre referál például, akit elégetnek.

" Másrészt nem is elégséges, mert ilyen alapon azt kellene mondanunk, hogy Pavlov kutyájának nemcsak az agyterülete, hanem a nyáladzása is az ételre referál ami abszurd. Hiszen, nincs releváns szempontból különbség az agyterület aktivitása és a nyáladzás között, mindkettőt az ételre való kondicionálás váltja ki."

Itt összezavarodtál. Ha Pavlov kutyájának valamely agyterületi aktivitásmintázata a nyáladzással jár együtt, akkor a nyáladzásra referál, ha az étellel, akkor az étellel. Feltételezhető, hogy a két minta nem azonos, bár az étel mintázata nyilván együtt jár a nyáladzás mintázatával, azaz tartalmazza az utóbbit.

"Ez is abszurd. A referálás az nem olyasmi, amit egy fizikai dolog csak úgy csinál magától. Ahhoz minimum asszociálódnia kell valamivel egy elmében, de még ez sem elég hozzá."

Szerinted abszurd. Szerintem az abszurd, amit te mondasz. Érvet nem mondasz, csak ismételgeted, hogy szubjektív idealistaként neked abszurd a monista elképzelés. De ez nem egy érv, ez csak a hited ismételgetése.

"A referálásnál nincs jelen, mert nem létezik olyan, hogy értelmező nélküli referálás."

De létezik. Például Pavlov kutyája kurvára nem értelmezi, hogy mire referál az agyaktivitása. Ettől még referál. Sőt, Pavlov kutyája adott esetben azt sem tudja, hogy az ételes tálja alumíniumból készült, de ettől még abból készült. Te olyan hülye vagy, hogy azt hiszed, hogy minden egyes élőlénynek tudnia kell mindenről, mert ha nem tud róla, akkor az nincs is.

Brendel Mátyás · http://ateistaklub.blog.hu/ 2017.05.05. 00:50:14

@brandon1: " Az ismerős példa lehet, hogy ha miután kipusztul az emberiség és a szél egy szó betűit fújja a homokba, az nem fog jelenteni már semmit, mert akkor az már csak egy egy természeti képződmény. Miért nem referál még akkor is arra, mint most, hacsak nem azért, mert akkorra már nincs ember, aki értelmezze?"

A betű az tényleg csak akkor refrál valamire, ha van, aki értelmezi. De az agyaktivitás az abban különbözik a betűtől, hogy a betű nem agy. A betű tényleg nem referál önmagában semmire, mert nem agyaktivitás. A betű nem információfeldolgozó rendszer, hanem jel.

Én sajnálom, hogy ennyire egyszerű dolgokat nem tudsz, de ez csak azt mutatja, mennyire debil vagy.

"Ez mitől hülyeség? Recsegés=hangérzet. Ha nincs megfigyelő, nincs hangérzet sem. "

A recsegés hang. Nem a hangérzetről, hanem a hangról beszéltem.

Te hülye vagy. A minimális eszedet csak a dolgok összekavarására használod, ahelyett, hogy a dolgok felfogására koncentrálnál. Ez idegesítő, mert ez egy olyan játék, amelyben te nagyon rossz vagy, azt, hogy rossz vagy benne azzal próbálod kompenzálni, hogy a vita játékszabályait felrúgod. És ahhoz is hülye vagy, hogy felfogd, mennyire vesztésre állsz.

Na msot persze azt ismerjük, hogy egy hülyegyerek akármeddig sakkozhat, és akárhány játszmát elveszthet, és minden vesztés után lesepri a sakktáblát, és "megmagyarázza", hogy de ő nyert. Ez a játék csak hülyegyerekeknek érdekes. Nekem megint unalmas vagy, és megint idegesítő az, hogy annyira hülye vagy, hogy fel sem fogod, milyen hülye vagy.

Brendel Mátyás · http://ateistaklub.blog.hu/ 2017.05.05. 01:00:57

@brandon1: Ezt a referálás és értelmezés dolgot össze fogod kavarni, amilyen hülye vagy.

Egy fa eldől az erdőben. Ez a fa természetesen akkor is reccsen, azaz akkor is képződik hang, a levegő akkor is remeg a fa kidőlésétől, ha senki nincs jelen. Ha X ember jelen van, akkor X hallja a reccsenést, azaz nemcsak hang képződik, hanem hangérzet is. (A hang és a hangérzet két külön dolog, te húgyagyú!).
Ha Y ember is jelen van, és látja, hogy X ember összerezzen a fa kidőlésére, miközben nem láthatja, akkor Y feltételezheti, hogy X hallotta a fadőlést, azaz Y értelmezheti azt, hogy X hallja a reccsenést. Ugyanakkor, ha Y nincs jelen, X akkor is hallja a reccsenést. Amikor X hallja a reccsenést, akkor X bizonyos agyterületében bizonyos aktivitásmintázat képződik. Ha ez a mintázat jól korrelál a reccsenéssel, akkor mondhatjuk azt, hogy ez az aktivitás reprezentálja a reccsenést, és a reccsenésre referál.

Ehhez nem kell, hogy Y jelen legyen, hogy ezt értelmezze, mert Y nélkül is van hang és van hangérzet. X ugyanis lehet akár egy kutya is. Ha az X kutya összerezzen, akkor az agyában nyilván referál valami a reccsenésre, emiatt rezzen össze. Ez független attól, hogy Y értelmezi-e. Téged az zavar meg, hogy ha X ember, akkor általában tudatában van annak, hogy reccsenést hall, azaz értelmezi is a saját referálását. De ez nem szükségszerű, és a kutya esetében nincs is meg.

Ahogy a fa kidőlése, és a reccsenés hangjának képződése is független attól, hogy az X kutya összerezzen-e.

Érted, te igenhülye?!

Brendel Mátyás · http://ateistaklub.blog.hu/ 2017.05.05. 01:10:29

@szemet: igen, ez is jó példa. egy számítógépes programban nyilvánvalóan szoktak lenni olyan változók, amelyeknek referenciája tök triviális, és tudjuk. például az autonóm autóban van egy radar érzékelő, és az algoritmusában van egy objektum-lista, amelynek minden egyes objektuma próbál egy valós tárgyra referálni. tudjuk, hogy mi, hogyan referál, tudjuk, hogy ennek mi az oka, hogy történik, tudjuk, hogy mi lehet a referálás következménye. tudjuk, hogy az objektum felbukkanása az objektum-listában a különféle hibák mellett nagyon jól korrelál az igazi tárgy jelenlétével, és a koordinátái jól korrelálnak az igaz tárgy koordinátávial. tudjuk, hogy ezeknek a változónak az algoritmusban mi felel meg, pontosan hol van tárolva a memóriában, milyen elektromos aktivitásmintázat felel meg neki.

ez az aktivitásmintázat referál, és ezt a referálást egyrészt értelmezhetik emberek, tudhatnak róla, érthetik, mert hiszen emberek készítették. de ha hirtelen minden ember meghal, és az autonóm autó mégis valahogy tovább megy még egy napig az utakon, akkor továbbra is referál, függetlenül attól, hogy értelmezi-e az ember, vagy nem.

brandon1 2017.05.05. 10:40:23

@Brendel Mátyás: @szemet: Az önvezető autó aktivitásmintázata vagy az idegi mintázat az agyban szerintetek referál. Ugye az, hogy x dolog referál y-ra, az egy tulajdonsága x-nek. De ti azt mondjátok, hogy bármilyen folyamat, mintázat, vagy dolog kizárólag egy információfeldolgozó rendszer részeként képes referálni, önállóan nem. Szóval akkor elismeritek, hogy az x dolognak (folyamat, mintázat)az ismert és mérhető fizikai tulajdonságain felül, azokra ráépülve, létrejön egy ilyen nem-fizikai, nem mérhető, "x referál y-ra" tulajdonsága is, azáltal, hogy egy ilyen rendszer részévé válik. Ez elég misztikusan hangzik.

"Egy fa eldől az erdőben. Ez a fa természetesen akkor is reccsen, azaz akkor is képződik hang, a levegő akkor is remeg a fa kidőlésétől, ha senki nincs jelen. Ha X ember jelen van, akkor X hallja a reccsenést, azaz nemcsak hang képződik, hanem hangérzet is."

A reccsenés nem hanghullám, hanem szubjektív hangérzet. Bizonyítás: ugyanazt a hanghullámot, amit mi reccsenésnek hallunk, lehet, hogy egy földönkívüli, a miénktől különböző hallószervével és agyával, egészen másnak hallaná, nem reccsenésnek, hanem mondjuk dörrenésnek vagy csobbanásnak, vagy bármilyen más hirtelen kialakuló hangérzete keletkezhet ugyanattól a hanghullámtól, amit mi reccsenésnek hallunk.

"De. Nagyon sok neurális korrelátumot igazoltak. Attól, hogy egy tájékozatlan hülye vagy, még nem lesz igazad." Mutass egy olyan mondatot abban a szócikkben, amit belinkeltél, ami ezt alátámasztja! Ne a majomkísérleteket, ne a tudatosság agyi korrelátumáról szóló fantáziálásokat, egyetlen IGAZOLT tényt arról, hogy az EMBERI érzékelési folyamatban egy adott típusú vizuális ingernek milyen neurális korrelátum felel meg! Mert én nem találok benne semmi ilyet. Amit találtam, az a materialisták hittétele: "The notion of a neural correlate of a mental state is an important concept for materialists, those philosophers and researchers who BELIEVE that all mental states are equivalent to brain states."

"Ha Pavlov kutyájának valamely agyterületi aktivitásmintázata a nyáladzással jár együtt, akkor a nyáladzásra referál, ha az étellel, akkor az étellel. Feltételezhető, hogy a két minta nem azonos" Miért feltételezed bizonyíték nélkül? És mi van, ha véletlenül mégis azonos: akkor ugye az van, hogy megcáfolja az "elméletedet".

Brendel Mátyás · http://ateistaklub.blog.hu/ 2017.05.05. 13:35:15

@brandon1:

" Szóval akkor elismeritek, hogy az x dolognak (folyamat, mintázat)az ismert és mérhető fizikai tulajdonságain felül, azokra ráépülve, létrejön egy ilyen nem-fizikai, nem mérhető, "x referál y-ra" tulajdonsága is, azáltal, hogy egy ilyen rendszer részévé válik. Ez elég misztikusan hangzik. "

Nem, ez egy fizikaira redukálható tulajdonság, le is írtam: korrelációnak kell lennie, és a rendszernek elég komplexnek kell lennie ahhoz, hogy referálásról beszéljünk.

"A reccsenés nem hanghullám, hanem szubjektív hangérzet.Bizonyítás: ugyanazt a hanghullámot, amit mi reccsenésnek hallunk, lehet, hogy egy földönkívüli, a miénktől különböző hallószervével és agyával, egészen másnak hallaná, nem reccsenésnek, hanem mondjuk dörrenésnek vagy csobbanásnak, vagy bármilyen más hirtelen kialakuló hangérzete keletkezhet ugyanattól a hanghullámtól, amit mi reccsenésnek hallunk."

Tehát neki a reccsenés hanghullám más hangérzetet kelt. Na bumm. Te most itt elkezdtél egy teljesen értelmetlen szematikai, definíciós vitát. Te beszélhetsz a "reccsenés" hangérzetről, és a reccsenés hangról beszéltem, nem a hangérzetről. A fogalmi kérdést tisztáztam, az értelmetlen vitádba nem megyek bele.

"Mutass egy olyan mondatot abban a szócikkben, amit belinkeltél, ami ezt alátámasztja!"

Kérlek:

"Using such design, Nikos Logothetis and colleagues discovered perception-reflecting neurons in the temporal lobe."

en.wikipedia.org/wiki/Neural_correlate

De egyébként a képalkotási technikák fejlődése óta tömkelegével vannak ilyen tudományos cikkek.

scholar.google.com/scholar?hl=hu&q=neural+correlate&btnG=

Ismétlem, attól, hogy tájékozatlan hülye vagy, illetve nehéz felfogású, még nem lesz igazad.

"Feltételezhető, hogy a két minta nem azonos" Miért feltételezed bizonyíték nélkül?"

Ez csak egy kiegészítő megjegyzés volt. Elmagyaráztam, hogy hogy van ez a fizikalista elképzelés szerint, mivel erre kérdeztél rá. A fizikalista elképzelésnek nyilván nem Pavlov kutyájának neurális korrelátumai a bizonyítékai, mivel ezt a kísérletet akkor még nem végezték el. Töménytelen más bizonyítékra viszont utaltam fentebb a neurális korrelátumoknál.

szemet 2017.05.05. 13:53:02

@brandon1: Bizony a referenciához kell valami információfeldolgozó rendszer, hiszen az egész referálás dolog ebben a kontextusban értelmezett, meg kell a referált dolog is. És ezek mind fizikai létezők, fizikai komponensekből állnak, amik önmagukban "ártalmatlannak" tűnnek ;), pl. atomokból.

en.wikipedia.org/wiki/The_Concept_of_Mind#Category_mistakes

" A much-cited example is of a foreign visitor being shown round Oxford (which has no campus) and after having been shown colleges, libraries, laboratories and playing fields asks in puzzlement "But where is the university?" The answer is, of course, all of these."

Atomic Warning 2017.05.06. 15:58:11

@brandon1: Te azt hiszed, hogy egy jelenségnek bármilyen tulajdonsága függ attól, hogy mi azt hogyan érzékeljük vagy hogyan tanuljuk meg kifejezni. egy jelenség nem változik attól ha azt valaki másképp érzékeli vagy másképp pl hibásan vagy más nyelven írja le. Az hogy valaki egy jelenséget hogyan ír le, az nem a jelenségtől függ, hanem az ő tudásától ami egy tanulási folyamat. Egy eltörő fa reccsenését elmondhatnánk úgy is, hogy a hangjelenség műszerrel rögzített görbéjét mutogatjuk, amit nem tudna értelmezni senki, csak pl ha behelyezi egy lejátszóba. ennek nem lenne semmi értelme. Amikor azt mondjuk hogy elreccsent a fa és kidőlt, az egy nem pontos leírás, de megfelel, mert kb el tudjuk képzelni, pedig lehet hogy amit mi erre az elmondásra elképzelünk, az teljesen más mint ami a valóságban lejátszódott. Emiatt a kettő nem függ össze, az hogy szerintünk ez és ez így történt, és ezt hogy fejezzük ki, az attól függ mit tudunk. Lehet hogy egy jelenségről nem tudunk eleget, és hülyeséget magyarázunk róla. Azt mondjuk pl a villámlásra hogy tegnap Zeusz mérges volt, pedig lófaszt volt mérges, csak villámlott, akkor is ha tudjuk hogy az pontosan micsoda, és akkor is ha nem. Ehhez nem szükséges az, hogy a villámlást vagy egy tetszőleges természeti jelenséget halál pontosan minden paraméterét tekintve leírjunk, mert ez pazarlás lenne és semmi értelme. Viszont azt is fel kellene fognod, hogy vannak a jelenségek vizsgálatára és rögzítésére műszereink, amik akkor is a jelenség valós tulajdonságait rögzítik ha nekünk ahhoz nincs vagy rossz az érzékszervünk, és ezeknek a műszeres méréseknek a kimenete teljesen alkalmas a jelenség pontos leírására, csak ezt kommunikálni a hétköznapokban nem éri meg akkora pontossággal mint amekkora pontossággal a műszer mér.

Példák: Azt mondom tegnap villámlott, vagy recsegve kidőlt egy fa. Azt hiszed ez elegendő ahhoz hogy mindenki tudja hogy tegnap villámlott és kidőlt egy fa. Ha viszont azt mondom, tegnap lefilmeztem ahogy villámlott és ahogy kidőlt egy fa, és ezt levetítem neked, akkor pontosabb képet kapsz a jelenségről. De mivel a kamera messze volt a fától, nem fogod pontosan hallani az hogyan reccsent. Ekkor eléd teszek egy kinyomtatott görbét amit a fa közelében vettem fel egy mikrofonnal, és azt mondom, látod, így recsegett a fa, nem úgy ahogy a kamera felvette távolról a szar mikrofonjával. Tényleg kretén vagy hogy ezeket nem látod és itt hablatyolsz a semmiről :)

Brendel Mátyás · http://ateistaklub.blog.hu/ 2017.05.12. 08:56:00

@brandon1: A blogpostban levő egyetlen érvre sem tudtál választ adni, csak engem bombáztál bugyuta "érvekkel". Kiemelnék a blogbostból egy olyan érvet, ami a referálásról szól, és amely számodra lesújtó:

"Hogy ez az érv miért rossz, és ezzel már Berkeley rendszerének durva hibáit is felhozom, az az, hogy Berkeley szerint léteznek elmék. Léteznek tehát különféle emberek. És amikor én azt mondom, hogy "George Berkeley, a híres filozófus", akkor az Berkeley szerint is egy olyan elgondolás, amely referál valamire, ami az én elmémen kívül van, méghozzá rá. Ha pedig én képes vagyok az elmémen kívül létező George Berkeley-ról beszélni, rá referálni, sőt, róla megtudni dolgokat, akkor ez a teáskanna esetében sem lehet fogalmilag problémás. Ezt az érvet Hylas nem veti fel, de ha felvetette volna, akkor Berkeley kénytelen lett volna elismerni, hogy a gondolataink referálhatnak az elmén kívüli dolgokra, mert ha más nem, akkor más személyek az elménken kívül vannak."

Ezt az érvet csak akkor tudod kikerülni, ha szolipszista vagy. Csakhogy a szolipszizmus nem tud magyarázatot adni arra, hogy a világ törvényszerűségei elég egyértelműen igazolnak külső realitásokra, mert a világban van rend, és vannak a kívánságaink ellen működő akadályok is.

Továbbá, ha szolipszista vagy, akkor ateista vagy, tehát vallás-ateizmus témában a mi oldalunkon állsz, még ha elég hülye módon is.

Ja, és ugye ha szolipszista vagy, akkor szerinted én nem létezem, tehát nem tudom, minek vitázol velem.:)

brandon1 2017.05.12. 18:56:28

@Brendel Mátyás: "Berkeley kénytelen lett volna elismerni, hogy a gondolataink referálhatnak az elmén kívüli dolgokra, mert ha más nem, akkor más személyek az elménken kívül vannak." Te alapvetően félreérted a szubjektív idealizmust. Berkeley amikor elmén kívüli dolgok nemlétezését vagy ellentmondásosságát állítja, nem a saját elméjében való létezésre gondol, hanem arra, hogy bizonyos dolgok minden elmétől függetlenül létezhetnek-e. Egyedül erre ad nemleges választ. Nemcsak hogy elismerte a saját elméjén kívüli szellemek (nem "dolgok"!) létezését, hanem egész filozófia azon a feltevésen alapul, hogy létezik Isten, aki egy végtelen szellem az ő (G. B.) elméjén kívül. A wikipedián pedig elolvashatod, hogy más elmék avagy szellemek létezésére szerinte nincs közvetlen bizonyítékunk, de jó okunk van hinni bennük.

"In contrast to ideas, a spirit cannot be perceived. A person's spirit, which perceives ideas, is to be comprehended intuitively by inward feeling or reflection (Principles #89). For Berkeley, we have no direct 'idea' of spirits, albeit we have good reason to believe in the existence of other spirits, for their existence explains the purposeful regularities we find in experience.[16] ("It is plain that we cannot know the existence of other spirits otherwise than by their operations, or the ideas by them excited in us", Dialogues #145). This is the solution that Berkeley offers to the problem of other minds."

Brendel Mátyás · http://ateistaklub.blog.hu/ 2017.05.12. 21:03:54

@brandon1: " Nemcsak hogy elismerte a saját elméjén kívüli szellemek (nem "dolgok"!) létezését, hanem egész filozófia azon a feltevésen alapul, hogy létezik Isten, aki egy végtelen szellem az ő (G. B.) elméjén kívül. "

Én is pontosan ezt mondtam, mi a faszt nem fogtál fel abból, hogy ez a legnagyobb cáfolata az érveinek?! "Dolog" alatt bármit értettem, ami létezik, tehát a szellemek is dolgok. De ha neked ez a szó nem tetszik, az nem változtat az érvem érvényességén.

Berkeley szerint is az elménk képes agyunkon kívüli "szellemekre" referálni. Azaz Berkeley is elismeri, hogy az elme képes referálni magán kívülre. Ezután sem ő, sem te nem érvelhettek azzal, hogy nem képes, mert elismeritek, hogy képes. Tökön szúrtátok magatokat. Nem kicsit, nagyon.

Brendel Mátyás · http://ateistaklub.blog.hu/ 2017.05.13. 11:01:39

@brandon1:

ilyen egyszerű ez:

1) Szerinted Berkeley létező szellem volt? Igen.
2) Leírtad azt a szót, hogy Berkeley? És ezzel a szóval a Berkeley nevű hajdan létezett filozófusra referáltál? Igen.
3) Ez a Berkeley nevű filozófus az elméden kívül volt? Igen.

Tehát el kell ismerned, hogy a te elméd képes az elméden kívül referálni.

Ennyi.

brandon1 2017.05.13. 16:28:48

@Brendel Mátyás:"Tehát el kell ismerned, hogy a te elméd képes az elméden kívül referálni." Ezt csak egy szolipszista nem ismerné el, Berkeley és én is elismerem. A szubjektív idealizmus azt tagadja, hogy léteznének dolgok az összes elmétől függetlenül (beleértve isten elméjét is). A kérdés az, hogyan elemezhető a "Létezett egykor a George Berkeley nevű filozófus" állítás. És ennek az elemzésében semmi olyan nincsen, ami vagy ne szellem, vagy ne idea lenne. Így értendő: Létezik G. B. elméje (amíg emberként élt, addig a mi világunkban létezett, most a túlvilágon létezik), és amíg a mi világunkban létezett, isten létrehozott ebben az elmében ideákat, amikor érzékelt, és isten létrehozott más emberek elméjében is ideákat, amik G. B. elméjének jelenlétére utaltak. Így például, amikor G. B. találkozott valakivel, isten létrehozta a G. B. testét és ruháját alkotó ideákat G. B. elméjében (mert G. B. érzékelte saját magát), és annak az elméjében is, akivel találkozott.
Berkeley szerint csak ez a kétféle típusú dolog létezik, elme vagy szellem, és idea. Az ideák létezéséről az érzékelés alapján tudunk. A szellemek létezéséről intuitív tudásunk van a saját elménk megfigyeléséből, más szellemek jelenlétére pedig közvetett bizonyítékunk van. Így alakítjuk ki a szellem fogalmát. És ez bizony Berkeley részéről nagyon keményen Ockham borotvájának alkalmazása mindenféle ezen túlmenő materialista fantazmagóriára, hogy létezik valamilyen materiális szubsztancia, ami állítólag hordozója az érzékelhető tulajdonságoknak, bár maga nem érzékelhető, mert absztrakt. Berkeley szerint nem lehet jó okot találni az anyag létezésének feltételezésére, erről szólnak az érvei. Ennyi. És ne keverd össze megint a szubjektív idealizmust a szolipszizmussal, mert nem azonos vele!

Brendel Mátyás · http://ateistaklub.blog.hu/ 2017.05.14. 01:34:05

@brandon1: 1) Nem kell folyton újra elmesélned, hogy Berkeley mit gondolt, mert végig hallgattam az egyik fő művét, szóval tudom. Az nem érvelés, hogy újra és újra elismételgeted, mit hitt. Az meg kifejezetten sunyiság, ha úgy teszel, mintha én nem tudnám.

2) Berkeley szubjektív idealizmusa, amit te is bizonyára képviselsz, mert nagyon véded hülyeség. Kifejtettem, hogy miért.

3) A szolipszista például a szkepticizmusában legalább következetes. A szolipszista is abból indul ki, hogy valójában minden empirikus tapasztalat az ő elméjében van. Tehát a legszűkebben véve, a legnagyobb igazoltsággal csak az elméjének létezése igazolt, és erre is szorítkozik.

4) A materialista és a dualista azt mondja, hogy az élményekbe olyan nagy fokú konzisztencia van, és olyan nagy fokú függetlenség, hogy ennek a megmagyarázásához érdemes feltételezni egy külső realitást. A dualista pluszban még azt is feltételezi, hogy a realitás két szubsztanciából áll, ami megoldhatatlan problémát jelent, ezt kifejtettem. Ez a két nézet az Occam borotvájával azért nem vágja le a külső realitást, mert az megmagyaráz valamit. Occam borotvája azt ugyanis nem mondja, hogy olyan feltételezéseket is el kell vetni, amelyek magyarázatul szolgálnak valamire.

5) A szubjektív idealista Occam borotváját tekintve már rosszabbul áll. Az igaz, hogy a sok külső anyagi tárgy helyett csak egy istent tételez fel, azaz primitív egyedszámlálást tekintve látszólag jobb, csak hát ebben az istenben benne van az egész anyag valóságnak megfelelő komplexitás, azaz komplexitást tekintve a szubjektív idealizmus bonyolultabb, hiszen abban az elmék komplexitása mellett az anyagi tárgyak komplexitása valójában benne van, csak ez a komplexitás istenbe van. És akkor még ehhez jön isten további komplexitása, hiszen istenben nem csupán az anyagi világ van benne komplexitásként.

6) A szubjektív idealizmus következetlen is, mert mindama érv, ami más elmék létezése mellett szól, az, vagy ahhoz hasonló érv szól az anyagi tárgyak létezése mellett. Például ha te azt hiszed, hogy én létezem, akkor erre az összes érved olyan, ami az interneten alapul. Az internetet a számítógépeden keresztül éred el. Azaz az összes érv, ami az én létezésem mellett szól, az a számítógéped létezése mellett is szól, és arra is ugyanolyan jó érv. Hülyeség tehát azt hinned, hogy én létezem, mert erre van 123 érved, de nem hinned a számítógéped létezésében, amelyre 123 érved van, plusz még egy csomó több érved, hiszen a számítógépedet tőlem függetlenül is látod, tapintod, hallod.

Ha te mégis azt hiszed, én létezem, de a számítógéped nem, akkor te hülye vagy. És Berkeley is viszonylag hülye volt, mert erre azért már ő is rájöhetett volna.

brandon1 2017.05.14. 10:01:40

@Brendel Mátyás: "minden empirikus tapasztalat az ő elméjében van. Tehát a legszűkebben véve, a legnagyobb igazoltsággal csak az elméjének létezése igazolt, és erre is szorítkozik." Ebben a szolipszista nem következetes. Az elméjében levő tapasztalat teljesen passzív, nem képes okozni semmit, az viszont nyilvánvalónak tűnik, hogy magát a tapasztalatot okozza valami, mert hiszen az érzékelés alapvetően passzív. A szolipszista szerint az ő saját elméje az, ami okozza az előtte levő alma látványának tapasztalatát. Lehetséges ez is, csak épp nagyon valószínűtlen és igazolhatatlan. Ennyiben az összes többi filozófia következetesebb, mert elfogadja ezt a megszorítást.

"A materialista és a dualista azt mondja, hogy az élményekbe olyan nagy fokú konzisztencia van, és olyan nagy fokú függetlenség, hogy ennek a megmagyarázásához érdemes feltételezni egy külső realitást" Nem értem, itt miért hagyod ki a szubjektív idealistákat, hiszen ők is ugyanezt mondják, azzal, hogy ezt a külső realitást istennek nevezik.

"Az igaz, hogy a sok külső anyagi tárgy helyett csak egy istent tételez fel, azaz primitív egyedszámlálást tekintve látszólag jobb" Akkor Ockham borotváját tekintve mégiscsak jobban áll, mert az lényegében a (szerinted) primitív egyedszámlálásról szól: „Csak szükség esetén posztulálj sokféleséget” hu.wikipedia.org/wiki/Occam_borotv%C3%A1ja

"azaz komplexitást tekintve a szubjektív idealizmus bonyolultabb, hiszen abban az elmék komplexitása mellett az anyagi tárgyak komplexitása valójában benne van, csak ez a komplexitás istenbe van" Nincsen istenben komplexitás. Benne nincsenek meg azok a "mechanizmusok", amiket a materialistának fel kell tennie az anyagi tárgyakban ahhoz, hogy magyarázni tudja, hogyan okozzák az érzeteket.
"És akkor még ehhez jön isten további komplexitása, hiszen istenben nem csupán az anyagi világ van benne komplexitásként." Nem jön hozzá, mert isten teljesen egyszerű. en.wikipedia.org/wiki/Divine_simplicity

"Ha te mégis azt hiszed, én létezem, de a számítógéped nem" Tévedsz. Berkeley soha nem tagadta a hétköznapi tárgyak létezését. "Berkeley does not deny the existence of ordinary objects such as stones, trees, books, and apples. On the contrary, as was indicated above, he holds that only an immaterialist account of such objects can avoid skepticism about their existence and nature. What such objects turn out to be, on his account, are bundles or collections of ideas."plato.stanford.edu/entries/berkeley/#3.1.1

Brendel Mátyás · http://ateistaklub.blog.hu/ 2017.05.16. 08:09:16

@brandon1:
"Ebben a szolipszista nem következetes. Az elméjében levő tapasztalat teljesen passzív, nem képes okozni semmit, az viszont nyilvánvalónak tűnik, hogy magát a tapasztalatot okozza valami"

Megismétled azt, amit én írtam. Csak primitívebb formában. Ilyen érvek, pontosabban megfogalmazott formában szólnak a szolipszizmus ellen. De ezek ugyanígy szólnak a szubjektív idealizmus ellen is. A szolipszista következetes, mert azt mondja, ez nem érdekli őt. A szubjektív idealista a következetlen, mert azt mondja, hogy ez érdekli őt (más elmék esetében), meg nem is érdekli (anyagi tárgyak esetében). Össze-vissza viselkedik.

" mert hiszen az érzékelés alapvetően passzív. A szolipszista szerint az ő saját elméje az, ami okozza az előtte levő alma látványának tapasztalatát. Lehetséges ez is, csak épp nagyon valószínűtlen és igazolhatatlan. Ennyiben az összes többi filozófia következetesebb, mert elfogadja ezt a megszorítást."

Nem. Ebbe a materializmus következetes, mert azt mondja, hogy az alma élményét az alma okozza. A szubjektív idealista a következetlen, mert azt mondja, hogy a másik ember élményét a másik ember okozza, de az alma elméjét nem az alma. Ez ostoba agyrém.

"Akkor Ockham borotváját tekintve mégiscsak jobban áll, mert az lényegében a (szerinted) primitív egyedszámlálásról szól: „Csak szükség esetén posztulálj sokféleséget” hu.wikipedia.org/wiki/Occam_borotv%C3%A1ja"

De ez primitív. Valójában nyilvánvalóan nem az érdekel minket, hogy hány létező van az elméletben, hanem az, hogy milyen bonyolult dolog az elmélet, mert attól függ, hogy mennyire gyorsan tudjuk átgondolni, és milyen könnyen tudjuk megjegyezni. Márpedig ezt várjuk az elmélettől. A létezők száma csupán a komplexitás approximációja.

De ráadásul nem is egyértelmű a dolog. Mert a materialista elképzelésben is mondhatjuk, hogy egyetlen dolog létezik: az Univerzum. És az Univerzumban ott van minden. A szubjektív idealizmusban meg istenben van minden. Mi a különbség? Az Univerzum nem más, mint a benne lévő dolgok (elmék is) összessége. A te istened viszont állítólag egy személy, és ez az extra komplexitás.

"Nincsen istenben komplexitás. Benne nincsenek meg azok a "mechanizmusok", amiket a materialistának fel kell tennie az anyagi tárgyakban ahhoz, hogy magyarázni tudja, hogyan okozzák az érzeteket."

Ha nincs benne, akkor nem tudja magyarázni. Akkor a szubjektív idealistával pont ugyanaz a baj, mint amit a szolipszistáról mondtál. Egyébként itt el is térsz Berkeley-tól, mert őszerinte benne van istenben a komplexitás.

Ha én látok egy lovat, akkor látok egy bizonyos lóformát. Ha meg is tapintom, akkor érzek egy ezzel korreláló formát. Továbbá a lóval gyakran együtt jár a nyerítés hangja, és a lócitrom szaga. A materialista ezt úgy magyarázza, hogy a ló egy állat, amelynek van teste, amely hangot ad, mert a lovak hordákban élnek, és kommunikálnak egymással, az evolúciós sikeresség miatt. A testet meg tudjuk tapintani, mert egy anyagi test, és a lócitrom szaglik, mert a ló anyagcseréjének termékeit tartalmazza. Ennek is megvan a biológiai magyarázata.

Te két dolgot tehetsz:

A) Vagy ugyanezeket a magyarázatokat, a hozzájuk szükséges komplexitással bele teszed istenbe, de akkor istenben is ott lesz ló, test, horda, biológia, és evolúció (ideája), meg minden, ami a materialista elméletben van. Attól, hogy istenben ezek nem materiálisan vannak benne, hanem idea-struktúraként, attól a komplexitásuk nem csökken.

B) Vagy ezeket nem magyarázod, akkor olyan vagy, mint a szolipszista.

"isten teljesen egyszerű."

egy teljesen egyszerű dolog nem lehet személy. egy személynek karaktere van (érzések, dolgok, amiket szeret, amiket nem szeret, céljai, vágyai). minden személynek van egy minimális karaktere, ami egy minimális komplexitás. ezek nélkül nem beszélhetünk személyről.

"Tévedsz. Berkeley soha nem tagadta a hétköznapi tárgyak létezését. "

Tudtommal tagadta a hétköznapi tárgyak létezését, ahogy azt a materialista érti. És én úgy fogalmaztam, a magam fogalmai szerint.

""Berkeley does not deny the existence of ordinary objects such as stones, trees, books, and apples. On the contrary, as was indicated above, he holds that only an immaterialist account of such objects can avoid skepticism about their existence and nature. What such objects turn out to be, on his account, are bundles or collections of ideas."plato.stanford.edu/entries/berkeley/#3.1.1"

Ilyen értelemben Berkeleynél is "léteznek" ezek a tárgyak, de akkor visszajutottunk oda, hogy ezek istenben vannak. Istenben lévő komplexitások. Sőt, most már megint úgy fogalmazol, hogy ezek a tárgyak létező dolgok. Azaz Occam borotvájánál össze kell számolnod, és akkor nem lesz kevesebb létező az elméletedben, hanem több.

Tehát a szubjektív idealizmus komplexebb, és épp az Occam borotvája elven is bukik.

Brendel Mátyás · http://ateistaklub.blog.hu/ 2017.05.16. 08:20:33

@brandon1: "Ebben a szolipszista nem következetes. Az elméjében levő tapasztalat teljesen passzív, nem képes okozni semmit, az viszont nyilvánvalónak tűnik, hogy magát a tapasztalatot okozza valami"

Megismétled azt, amit én írtam. Csak primitívebb formában. Ilyen érvek, pontosabban megfogalmazott formában szólnak a szolipszizmus ellen. De ezek ugyanígy szólnak a szubjektív idealizmus ellen is. A szolipszista következetes, mert azt mondja, ez nem érdekli őt. A szubjektív idealista a következetlen, mert azt mondja, hogy ez érdekli őt (más elmék esetében), meg nem is érdekli (anyagi tárgyak esetében). Össze-vissza viselkedik.

" mert hiszen az érzékelés alapvetően passzív. A szolipszista szerint az ő saját elméje az, ami okozza az előtte levő alma látványának tapasztalatát. Lehetséges ez is, csak épp nagyon valószínűtlen és igazolhatatlan. Ennyiben az összes többi filozófia következetesebb, mert elfogadja ezt a megszorítást."

Nem. Ebbe a materializmus következetes, mert azt mondja, hogy az alma élményét az alma okozza. A szubjektív idealista a következetlen, mert azt mondja, hogy a másik ember élményét a másik ember okozza, de az alma elméjét nem az alma. Ez ostoba agyrém.

"Akkor Ockham borotváját tekintve mégiscsak jobban áll, mert az lényegében a (szerinted) primitív egyedszámlálásról szól: „Csak szükség esetén posztulálj sokféleséget” hu.wikipedia.org/wiki/Occam_borotv%C3%A1ja"

De ez primitív. Valójában nyilvánvalóan nem az érdekel minket, hogy hány létező van az elméletben, hanem az, hogy milyen bonyolult dolog az elmélet, mert attól függ, hogy mennyire gyorsan tudjuk átgondolni, és milyen könnyen tudjuk megjegyezni. Márpedig ezt várjuk az elmélettől. A létezők száma csupán a komplexitás approximációja.

De ráadásul nem is egyértelmű a dolog. Mert a materialista elképzelésben is mondhatjuk, hogy egyetlen dolog létezik: az Univerzum. És az Univerzumban ott van minden. A szubjektív idealizmusban meg istenben van minden. Mi a különbség? Az Univerzum nem más, mint a benne lévő dolgok (elmék is) összessége. A te istened viszont állítólag egy személy, és ez az extra komplexitás.

"Nincsen istenben komplexitás. Benne nincsenek meg azok a "mechanizmusok", amiket a materialistának fel kell tennie az anyagi tárgyakban ahhoz, hogy magyarázni tudja, hogyan okozzák az érzeteket."

Ha nincs benne, akkor nem tudja magyarázni. Akkor a szubjektív idealistával pont ugyanaz a baj, mint amit a szolipszistáról mondtál. Egyébként itt el is térsz Berkeley-tól, mert őszerinte benne van istenben a komplexitás.

Ha én látok egy lovat, akkor látok egy bizonyos lóformát. Ha meg is tapintom, akkor érzek egy ezzel korreláló formát. Továbbá a lóval gyakran együtt jár a nyerítés hangja, és a lócitrom szaga. A materialista ezt úgy magyarázza, hogy a ló egy állat, amelynek van teste, amely hangot ad, mert a lovak hordákban élnek, és kommunikálnak egymással, az evolúciós sikeresség miatt. A testet meg tudjuk tapintani, mert egy anyagi test, és a lócitrom szaglik, mert a ló anyagcseréjének termékeit tartalmazza. Ennek is megvan a biológiai magyarázata.

Te két dolgot tehetsz:

A) Vagy ugyanezeket a magyarázatokat, a hozzájuk szükséges komplexitással bele teszed istenbe, de akkor istenben is ott lesz ló, test, horda, biológia, és evolúció (ideája), meg minden, ami a materialista elméletben van. Attól, hogy istenben ezek nem materiálisan vannak benne, hanem idea-struktúraként, attól a komplexitásuk nem csökken.

B) Vagy ezeket nem magyarázod, akkor olyan vagy, mint a szolipszista.

"isten teljesen egyszerű."

egy teljesen egyszerű dolog nem lehet személy. egy személynek karaktere van (érzések, dolgok, amiket szeret, amiket nem szeret, céljai, vágyai). minden személynek van egy minimális karaktere, ami egy minimális komplexitás. ezek nélkül nem beszélhetünk személyről.

"Tévedsz. Berkeley soha nem tagadta a hétköznapi tárgyak létezését. "

Tudtommal tagadta a hétköznapi tárgyak létezését, ahogy azt a materialista érti. És én úgy fogalmaztam, a magam fogalmai szerint.

""Berkeley does not deny the existence of ordinary objects such as stones, trees, books, and apples. On the contrary, as was indicated above, he holds that only an immaterialist account of such objects can avoid skepticism about their existence and nature. What such objects turn out to be, on his account, are bundles or collections of ideas."plato.stanford.edu/entries/berkeley/#3.1.1"

Ilyen értelemben Berkeleynél is "léteznek" ezek a tárgyak, de akkor visszajutottunk oda, hogy ezek istenben vannak. Istenben lévő komplexitások. Sőt, most már megint úgy fogalmazol, hogy ezek a tárgyak létező dolgok. Azaz Occam borotvájánál össze kell számolnod, és akkor nem lesz kevesebb létező az elméletedben, hanem több.

Brendel Mátyás · http://ateistaklub.blog.hu/ 2017.05.16. 08:21:12

@brandon1: "What such objects turn out to be, on his account, are bundles or collections of ideas."

És ezek az ideakötegek, ezek egyrészt az emberek fejében is részben benne vannak, már amennyit az emberek tudnak a tárgyakról, másrészt pedig a teljes ideakötegek benne kellenek, hogy legyenek isten elméjében, aki ugye irányítja azt, hogy mi milyen tapasztalatokat kapunk. A szubjektív idealizmusban ugyanis a tapasztalataink nem a saját hallucinációink, hanem isten vetíti nekünk az Univerzum szimulált valóságát. Csakhogy akkor ehhez isten elméjében kell, hogy legyen a tárgyak teljes leírása, a szimulált Univerzum teljes adattömege, amely pontosan annyi komplexitást jelent, mint amilyen a materialista Univerzum. Isten elméjében ugyanis a szimulált univerzumon túl is kell, hogy legyen valami, hogy elmondhassátok, hogy isten egy karakterrel, egyéniséggel rendelkező személy.

Röviden: az Univerzum komplexitása legyen U. Függetlenül attól, hogy ez egy materiális univerzum, vagy ideaköteg isten elméjében, ez az U komplexitáson nem változtat.

A materialista elképzelés komplexitása U.

A szubjektív idealista elképzelés komplexitása U+I, ahol I isten komplexitása az U-n túl (személyiség, egyéniség, karakter). Ha I=0, akkor nincs isten.

Tehát a szubjektív idealizmus komplexebb, és épp az Occam borotvája elven is bukik.

szemet 2017.05.16. 10:50:51

@Brendel Mátyás:

A tudatainkban sosincs benn az egész materializmus által posztulált világ, csak töredéke. Pl. ahhoz hogy lássuk a Jupiter nagy vörös foltját nem kell atomi szinten modellezni, mondjuk elég lehet csak statisztikus gáztörvényekkel, sőt általában csak elegendő belevetíteni a képét a távcsövünkbe, ha épp belenézünk stb...

A kérdés hogy egy ilyen Isten egyszerűbb vagy bonyolultabb Occam elv alapján?

Szerintem bonyolultabb, de én az - Occam elv nem létezők száma hanem modell komplexitása lapján való használatát tartom jogosnak.

Egy ismert bevett analóg példa az érthetőség kedvéért a kvantummechanikából:

pl. a koppenhágai értelmezés komplexebb mint a sokvilág elmélet mert bevezet egy plusz mechanizmust a modellbe (hullámfüggvény összeomlása, és ennek körítése - mitől, mikor, miért?), a sokvilág elmélet egyszerűen kihagyja ezt, anélkül hogy sérülne az empíria, tehát egyszerűbb a modell hiába több a létező.

Brendel Mátyás · http://ateistaklub.blog.hu/ 2017.05.16. 20:58:25

@szemet: a mi tudatunkban valóban nincs benne a világ teljes komplexitása. de mindegy is, hogy a tudatunk tartalma a materializmusnál és a szubjektív idealizmusnál is egyaránt van, létezik, és ugyanaz, ugyanolyan a komplexitása.

ami isten tudatát illeti, a materializmusban ugye nincs, a komplexitása tehát nulla. a szubjektív idealizmusban azonban isten szimulálja a teljes világot, azaz isten fejében egy pontosan ugyanolyan komplexitású modellnek kell lennie, mint ami a materializmus szerint az Univerzum teljes elmélete. és mivel le is szimulálja, ezért a teljes Univerzum is benne van isten elméjében.

ehhez jön még isten plusz komplexitása, mivel van még személyisége is, például egy ideig imádta a zsidókat, meg ilyenek.:)

brandon1 2017.05.17. 20:17:49

@Brendel Mátyás: Nem értem az összehasonlításodat. A szubjektív idealizmus szimulált világának a komplexitása szerintem végtelenül kisebb, mint az anyagi világ komplexitása a materialista felfogás szerint. Isten elég, ha az univerzumnak az általunk tapasztalt részét szimulálja. Vagyis például elég, ha csak annyi csillagot szimulál, amennyit az égen látunk + még annyit, amennyit (űr)távcsővel "látni" lehet. Szabad szemmel kb. 10 ezer csillag látható. De még ha ennek a százszorosát is lehet műszerekkel látni, az a szám is végtelenül eltörpül a l0^19 mellett. A materializmus szerint az ősrobbanás óta ennyi csillag keletkezett, és ez csak egy durva becslés.

szemet 2017.05.17. 20:43:12

Pont ez mondtam. Ha valami létezését a megfigyelés befolyásolja az egy komplexebb modell mintha megfigyelőfüggetlenül állandó lenne.

Ha megfigyelés (mérés) összeomlasztja a hullámfügvényt az egy komplexebb modell szemben azzal ha nincs ilyen befolyásolás - még ha több is a létező. Ha a tej megjelenik ha kinyitom a hűtőt és eltűnik ha becsukom az a tejnek egy komplexebb modellje mint egy folytonosan létező tej - még ha szumma ez több létezőt is jelent.

Egy olyan Isten/világegyetem modellje ami minimális számú létezővel tartja konzisztensen a tudatokat egyszerűen komplexebb mint egyszerűen időben folytonosnak tételezni az amúgy annak tűnő létezőket.

Ha nem a létezők számát tekinted az Occam elv lapjának, hanem a modellek nyakatekertségét (én pl. ezt tartom értelmesnek), egy ilyen "lazy-evaluator" Isten sokkal komplexebb.

brandon1 2017.05.17. 23:05:41

@szemet: Igen de a tejes példáddal ellentétben az idealista modellben istennek a számunkra nem tapasztalható valóságot - csillagokat, stb - nem kell fenntartania, semmilyen módon. Az univerzum távolabbi, számunkra nem észlelhető részeinek létezésére itt nincs szükség. A materialistának viszont a felfoghatatlan számú csillagot időben folytonosnak kell tételeznie, természetesen, nem mint tényleges tapasztalatot, hanem mint az észlelés lehetőségét biztosító komplex anyagi létezőket. Az időtényezőről nem is beszélve, értve ezalatt az ember megjelenéséig eltelt időt.

Brendel Mátyás · http://ateistaklub.blog.hu/ 2017.05.17. 23:39:17

@brandon1: "A szubjektív idealizmus szimulált világának a komplexitása szerintem végtelenül kisebb, mint az anyagi világ komplexitása a materialista felfogás szerint. Isten elég, ha az univerzumnak az általunk tapasztalt részét szimulálja. Vagyis például elég, ha csak annyi csillagot szimulál, amennyit az égen látunk + még annyit, amennyit (űr)távcsővel "látni" lehet. Szabad szemmel kb. 10 ezer csillag látható. De még ha ennek a százszorosát is lehet műszerekkel látni, az a szám is végtelenül eltörpül a l0^19 mellett. A materializmus szerint az ősrobbanás óta ennyi csillag keletkezett, és ez csak egy durva becslés. "

Nem elég. Több szempontból sem elég.

1) Honnan tudja isten azt, hogy mi mit fogunk megnézni?
2) Honnan tudod, hogy csak mi, emberi elmék vagyunk szubjketumok?
3) A mi megismerésünk is túl megy a tapasztalatainkon, és potenciálisan végtelenül túl megy.

Például van egy óra ránézek 12:00-kor. Ezek után elszámolok másodpercenként egyesével magamban 60-ig, és ránézek az órára. Amennyire pontos a számolásom, az órának 12:01-et kell mutatnia.

Istennek az óra teljes működését le kellett szimulálnia 12:00-tól 12:01-ig. És ez a köztes szimuláció egyszerűnek tűnhet, de van, amikor a 12:00 és 12:01 közötti folyamat nagyon bonyolult lehet. Például méréseket végezünk a világ meteorológiai állomásain 2018. január 1-én 12:00-kor és egy bazi nagy számítógép szimulációt végez, és megjósolja az időjárást 2018. január 1. 12:00 és 2019. január 5 12:00 óra között. Ezek után pedig a meteorológiai állomások rendszeresen végeznek méréseket, és az egész világ összes embere rendszertelenül végez méréseket. És ezek a mérések nagyjából megfelelnek a szimulációnak. Nem nagyon térnek el tőle. Ez pedig azt jelenti, hogy isten nem definiálhatott kénye-kedve szerint akármilyen időjárást, hanem kénytelen volt leszimulálni az egész Föld időjárását, mert egyszerűbben ezt nem tudom, hogy kaphatta volna meg. És mivel az emberi megismerés potenciálisan bármire képes, ezért istennek végül is az a legbiztosabb, hogy mindent le kell szimulálnia. Ha valamit kihagy, sose tudhatja, hogy valamikor valamelyik ember nem tud-e visszakövetkeztetni rá, és nem fedezi fel a hiányosságot, mint inkonzisztenciát isten szimulációjában.

Általánosítva: mivel a mi megismerésünk túl megy a tapasztalatainkon, és pontosan azért gondoljuk, hogy a szolipszizmus nem elég, mert ezen tapasztalatok mögött egy konzisztenciával, állandósággal, szabályossággal rendelkező realitás van, ezért istennek egy természettörvények szerint működő univerzumot kell szimulálnia. Ezt nem nagyon tudja máshogy, mint úgy, hogy egy természettörvénnyel rendelkező univerzumot végigszimulál pontosan azon természettörvények alapján, amelyekről azt akarja elhitetni, hogy azok a természettörvények.

Ja, egyébként egy másik érv jutott eszembe. Ugyan mi a fasznak csinálná isten ezt az átverő s dolgot, ha igazi univerzumot is teremthetne?!

Brendel Mátyás · http://ateistaklub.blog.hu/ 2017.05.17. 23:41:48

@szemet: szerintem a helyes ér az, hogy ha te beteszed a tejet a hűtődbe, és kiveszed 3 nap múlva a tejet a hűtőből, akkor istennek le kell szimulálnia, hogy mi történt vele közben a hűtőben, például azt, hogy megromlott-e. muszáj leszimulálnia a köztes részt is, hogy tudja, milyen tejet veszel ki a hűtőből.

Brendel Mátyás · http://ateistaklub.blog.hu/ 2017.05.17. 23:43:01

@brandon1: isten soha nem tudhatja, hogy mi nem tudunk-e meg valahogy mindenféle dolgokat a legtávolabbi csillag történetéből, hiszen Julius Caesar életéről is sokat tudunk, holott egyikünk sem látta.

brandon1 2017.06.28. 20:43:41

@Brendel Mátyás: "mivel a mi megismerésünk túl megy a tapasztalatainkon"

Ami túlmegy mindenféle lehetséges tapasztalaton, az nem megismerés, mert igazolhatatlan. Csak hinni lehet benne, ellenőrizni nem lehet.

"és pontosan azért gondoljuk, hogy a szolipszizmus nem elég, mert ezen tapasztalatok mögött egy konzisztenciával, állandósággal, szabályossággal rendelkező realitás van"

Nem a realitás ilyen, hanem a tapasztalataink ilyenek. A le
legfontosabb kérdés pedig éppen az, hogy mi az eredete ezeknek a tulajdonságoknak. És ezen a ponton a teista hipotézis sokkal jobb magyarázat, mint a materialista, mert érthető magyarázatot ad a tapasztalatnak ezekre a tulajdonságaira, vagyis a konzisztenciára, állandóságra, szabályosságra. A materialista szerint a soha meg nem tapasztalható anyagi szubsztancia a természettörvények szerint viselkedik, amik ugyan az említett tulajdonságokkal rendelkeznek, de a létezésükre nem adható magyarázat. A teista szerint viszont az említett tulajdonságokra a magyarázat az isteni szándék, az akarat állandósága, amit az ember a saját akaratának a tapasztalatából ismerhet, azokból az esetekből, amikor valamit konzisztensen és szabályszerűen csinál. Vagyis, az isteni szándékosság és intelligencia létezésének a feltételezése érthetővé teszi a természettörvényeket, amiket a materializmus kénytelen végső megmagyarázhatatlan adottságokként elfogadni. Az összes többi ellenvetésedet fölösleges akadékoskodásnak tartom, mert ha isten létezik, akkor Ő mindenható és mindentudó, ezért előre látja a jövőt is.

Brendel Mátyás · http://ateistaklub.blog.hu/ 2017.06.28. 23:01:44

@brandon1:

"Ami túlmegy mindenféle lehetséges tapasztalaton, az nem megismerés, mert igazolhatatlan. Csak hinni lehet benne, ellenőrizni nem lehet."

Én a "túl megy" kifejezést semmiképpen nem úgy értettem, hogy az igazolhatatlanságig túl megy, mert nincs kapcsolata a tapasztalatokkal, hanem csak úgy, hogy túl megy a merő fenomenalizmuson, amely szerint csak tapasztalatok vannak, oszt ennyi. Ez elég kézenfekvő, sajnálom, hogy hülye vagy, hogy felfogd, és magyarázni kellett.

"Nem a realitás ilyen, hanem a tapasztalataink ilyenek."

Itt olyannal vitázol, amit nem állítottam. Mi elsőre azt látjuk, hogy a tapasztalataink ilyenek, ezalapján feltételezzük, hogy a realitás ilyen. Ez elég kézenfekvő, sajnálom, hogy hülye vagy, hogy felfogd, és magyarázni kellett.

"És ezen a ponton a teista hipotézis sokkal jobb magyarázat, mint a materialista, mert érthető magyarázatot ad a tapasztalatnak ezekre a tulajdonságaira, vagyis a konzisztenciára, állandóságra, szabályosságra."

Az egész postom arról szól, és erős érveket ad amellett, hogy a faszt.

" A materialista szerint a soha meg nem tapasztalható anyagi szubsztancia a természettörvények szerint viselkedik, amik ugyan az említett tulajdonságokkal rendelkeznek, de a létezésükre nem adható magyarázat."

A tapasztalatok magyarázatát nyilván nem más tapasztalatokban keressük, és a magyarázat magyarázatát egy idő után már nyilván nem kérdezhetjük meg, mert ez végtelen regresszus. Itt tehát ostoba elvárásaid vannak. A magyarázati séma a nomologikusdeudktív modell, ebben várjuk a magyarázatokat, mert ez az, ami par excelelnce magyarázatnak gondolunk. A teista "magyarázatok" egyébként még kevésbé felelnek meg az igényeidnek. isten nem érzékelhető, és istennek nincs magyarázata. De ráadásul még a magyarázat konzisztenciája, logikája is förtelmesen gyenge.

Ezt demonstrálja a tipikus hívő szófordulat: "Isten útjai kifürkészhetetlenek".

" A teista szerint viszont az említett tulajdonságokra a magyarázat az isteni szándék, az akarat állandósága, amit az ember a saját akaratának a tapasztalatából ismerhet, azokból az esetekből, amikor valamit konzisztensen és szabályszerűen csinál. Vagyis, az isteni szándékosság és intelligencia létezésének a feltételezése érthetővé teszi a természettörvényeket, amiket a materializmus kénytelen végső megmagyarázhatatlan adottságokként elfogadni."

Csak tudod, az a nagy büdös helyzet, hogy az emberi szándékok és akarat kurvára nem következetes, nagyon labilis, sokszor kifürkészhetetlen, és bizony anyagi körülményektől, és természettörvényektől függ. Pont fordítva van: nem az ember jelleme az, ami a természettörvényeket magyarázza, hanem az ember szánalmas kis jellemének hóbortjait magyarázzák egészen jól a természettörvények. Azt is nagyon jól tudjuk, hogy személy, karakter, jellem, akarat nincs anyagi hordozó nélkül, nem is képzelhető el nélküle. És mindig ez az anyagi hordozó magyarázza a jellemet. Amikor Józsi megöli Julcsit, akkor a magyarázatként nem fogadjuk el azt, hogy: "mert ezt akarta, punkt um", magyarázatként mindig azt fogadjuk el, ami Jószi evilági, materiális élettörténetéből és a materiális körülményekből adódó természeti azokra épülő társadalmi törvényekből adódik.

Még egyszer mondom, a magyarázat sosem az, hogy: "mert Józsi ezt akarta, fogadd már el, te hülye, és ne kérdezősködj!"

Ezzel szemben nagyon jó magyarázatnak fogadjuk el azt, hogy ha valaki rámutat, hogy Józsinak tetszett Julcsi, de Julcsi nem akarta Józsit, Józsi otthon erőszakos környezetben nőtt fel, otthon is ezt látta, és hasonlók, egészen esetleg a pszichológiai,genetikai és neurobiológiai magyarázatokig. Ez magyarázat faszfejkém, az nem magyarázat, hogy: "Mert Józsi ezt akarta, ne kérdezz többet, kuss!"

Itt ezt már kifejtettem bővebben:

ateistaklub.blog.hu/2013/09/20/miert_nem_lehet_szemely_a_magyarazat

Te egy nagy kibaszott semmin alapuló valamilyen szellemet képzelsz el, amely szellem karaktere a levegőben lóg, és nincs magyarázata.

Arról nem beszélve, hogy egyetlen vallás sincs, amely ténylegesen akár csak vázlatosan is tudna arra választ adni, hogy mi a jó kurva istent akart isten a világgal, mi a szándéka, miért teremtette olyanra, amilyen, és ez hogyan segíti a szándékában. A Bibliában lehet, hogy neked meglepő módon, de erről egy kurva büdös szó nincs. Egy kukkot sem mondanak arról, mi is isten szándéka a világgal, mi a jó kurva anyádnak teremtette.

Úgyhogy elmész a jó büdös nénikédbe az ostoba, meggondolatlan, hazug baromságaiddal!

Az összes többi ellenvetésedet fölösleges akadékoskodásnak tartom, mert ha isten létezik, akkor Ő mindenható és mindentudó, ezért előre látja a jövőt is.

brandon1 2017.06.29. 20:55:27

@Brendel Mátyás: "A magyarázati séma a nomologikusdeudktív modell, ebben várjuk a magyarázatokat" Épp a modellnek a nomologikus része marad rejtély. Miért léteznek a természettörvények, miért használhatjuk őket az egyes jelenségek magyarázatára? erre nincs válasz a materialista világképben. A teista hipotézis viszont magyarázza.

"A tapasztalatok magyarázatát nyilván nem más tapasztalatokban keressük" A tapasztalatok eredetének és rendszerességének magyarázatára egy kitűnő hipotézis az isteni szándék, mert . Mi a bajod vele, talán az, hogy ?

"az a nagy büdös helyzet, hogy az emberi szándékok és akarat kurvára nem következetes, nagyon labilis, sokszor kifürkészhetetlen" Természetesen nem végeztél vizsgálatot a sok milliárd ember különféle viselkedését illetően, ezért amit írsz az egyrészt megalapozatlan, másrészt nem az ilyen emberi szándék a mintája az isteni szándékosságnak, hanem az idealizált, emberi gyengeségektől és fogyatékosságtól mentes emberi természetet kell alapul venni a párhuzamhoz. Vannak példák az emberi állhatatosságra és kitartásra, a történelemben, mint amilyenek a nagy emberek pozitív példái.

"A Bibliában lehet, hogy neked meglepő módon, de erről egy kurva büdös szó nincs." Hazudsz, mert van róla szó a Bibliában. In general, Scripture teaches us that God created the world and all that is in it for His own glory and because He desired to share His life with others. The creation of all these things demonstrates His glory, His love, grace, mercy, wisdom, power, goodness, etc. Compare Psalm 19:1; 8:1; 50:6; 89:5.

brandon1 2017.06.29. 21:02:48

@Brendel Mátyás: "Mi elsőre azt látjuk, hogy a tapasztalataink ilyenek, ezalapján feltételezzük, hogy a realitás ilyen. Ez elég kézenfekvő" Ami neked vagy másoknak a hétköznapokban kézenfekvő, az egyáltalán nem biztos, hogy igaz is. A gyerekek és a hétköznapi gondolkodás számára kézenfekvő a dualizmus: www.edge.org/conversation/paul_bloom-natural-born-dualists De ha a materializmus igaz lenne, akkor semmi jelentősége nem volna ennek a ténynek.
A másik baj ezzel, hogy azt állítod, hogy az anyag állandó és stabil és és rendszeres okozója a tapasztalatnak. Ami viszont körbenforgáshoz vezet, mert először az anyag stabilitására a tapasztalat rendszerességéből következtetsz, de aztán a tapasztalat rendszerességét magyarázod az anyag állandóságára, vagyis elmétől független létezésére hivatkozva (amit per definitionem nem tapasztalhatsz).

Brendel Mátyás · http://ateistaklub.blog.hu/ 2017.06.30. 09:40:39

@brandon1:

"Épp a modellnek a nomologikus része marad rejtély. Miért léteznek a természettörvények, miért használhatjuk őket az egyes jelenségek magyarázatára? erre nincs válasz a materialista világképben. A teista hipotézis viszont magyarázza."

de mi a fasznak nem bírod elolvasni, amit küldtem neked linket, te hülye barom?!

1) természettörvények nem léteznek, hanem érvényesek
2) ha megkérdezed, hogy miért érvényesek a természettörvények, akkor mi a faszt vársz magyarázatnak? mi a fasz lenne képes ezt magyarázni?
3) akármi is a magyarázat, csak végtelen regresszushoz jutsz, mert ha a természettörvények X miatt érvényesek, akkor a follow up question: de miért létezik X?

4) például, ha azt mondod, a természettörvények isten miatt vannak, akkor nem is egy további kérdés merül fel, hanem több:

a) miért létezik isten?
b) mi igazolja, hogy isten a természettörvények magyarázata, és nem más? ugyanis erre sincs független igazolásod.
c) miért pont ezeket a természettörvényeket akarja isten?
d) hogy a jó büdös francba tud ez az isten természettörvényeket teremteni?

tehát 1 kérdés megválaszolása érdekében egy igazolhatatlan hipotézist tettél fel, amely 4 új kérdéshez vezet.

ostoba vagy.

ráadásul az emberi analógiád sem jó, ugyanis az ember csak társadalmi törvényeket tud teremteni, amelyek konszenzuálisak és nem áthághatóak. azt tudjuk, hogy a társadalmi törvényeket az ember hogy egyezik meg, és hogyan próbálja betartatni, de azt tudjuk, hogy nem áthághatatlanok. a természettörvények egészen más jellegűek.

" Természetesen nem végeztél vizsgálatot a sok milliárd ember különféle viselkedését illetően"

de végeztem, végeztünk.

"nem az ilyen emberi szándék a mintája az isteni szándékosságnak, hanem az idealizált, emberi gyengeségektől és fogyatékosságtól mentes emberi természetet kell alapul venni a párhuzamhoz."
huzamodat, mint alátámasztó érv. mert az ugyan nem valószínűsít semmit, hogy: bizonyára van isten, mert az ember is ilyen és ilyen, várj, az ember sem ilyen és ilyen, de egy hipotetikus, nem létező ember lehetne ilyen.

ez olyan hülye indoklás, mintha azzal jönnél, hogy: "a Balatonban örök élet vize kell, hogy legyen, hiszen a fürdőkádamban is az van, igaz, a fürdőkádamban sincs, de egy idealizált fürdőkádban az lenne".

"A Bibliában lehet, hogy neked meglepő módon, de erről egy kurva büdös szó nincs." Hazudsz, mert van róla szó a Bibliában. In general, Scripture teaches us that God created the world and all that is in it for His own glory and because He desired to share His life with others. The creation of all these things demonstrates His glory, His love, grace, mercy, wisdom, power, goodness, etc. Compare Psalm 19:1; 8:1; 50:6; 89:5. "

hazudsz. ezen zsoltárok egyike sem állítja, hogy isten mi célból teremtette az Univerzumot. a legtöbbet a 19. zsoltár állít, de csak annyit, hogy: "Az egek beszélik Isten dicsőségét, és kezeinek munkáját hirdeti az égboltozat."

Ez, és a többi zsoltár is csak annyit állít, hogy az Univerzum milyen tökéletes szerinte ilyen, olyan, és amolyan szempontból. ez egy argumentum from design. De, hogy isten miért teremtette az Univerzumot, arról egy szó sincs. Azt sem állítják, hogy azért teremtette, mert tetszik neki, vagy ilyesmi. De ez egyébként elég gyenge cél lenne, és, ahogy mondtam, felveti a következő kérdéseket:

1) miért tetszik istennek pont egy ilyen univerzum?
2) hogy teremtette
3) mi az, ami munkál istenben, ami meghatározza a tetszéseit?

+1) ráadásul tudjuk, hogy tökéletes az Univerzum a fenét! istennek tényleg tetszik az, hogy egy kéjgyilkos, terrorista szervezetek gyerekeket kínoznak, vagy egy természeti katasztrófában gyerekek szörnyű halált halnak?

továbbá, ha itt az ember analógiáját vesszük, akkor azt mondhatjuk, hogy az embereknek az tetszik, ami az evolúció szerint érdeke (enni, inni, kefélni, stb.), és/vagy, amit belé neveltek (a Polgár lányok szeretnek sakkozni, mert az apjuk beléjük nevelte), vagy, amire valamilyen más okból az életük során rákattantak (például Józsi szeret cigizni, mert egyszer függő lett). ezek a különféle okok nyilván kombinálódhatnak, és ezen kívül a konkrét ok a jelen időben mindig az agyban kódolt preferencia. már kérdeztem, hogy az emberi és a technikai analógia szerint minden ágens egy hardveren fut, milyen hardveren fut isten?

istennél tehát mindez sehol sincs meg. egy kérdés magyarázatára egy teljesen légből kapott, igazolhatatlan hipotézist tettél fel, aminek se hatásmechanizmusa, és sokkal több kérdést vet fel, mint amit megmagyaráz.

Brendel Mátyás · http://ateistaklub.blog.hu/ 2017.06.30. 09:52:08

@brandon1: nincs körkörösség. minden magyarázatot két irányba lehet nézni:

A magyarázza B-t.

B-ből A-ra következtetek, mint hipotézis

ez minden magyarázatnál így van, és nem körkörösség, mert a logikai dedukció mindig A => B irányú. és mindig B az, amiről először tudunk, és A az, amire visszakövetkeztetünk a dedukció alapján.

az más kérdés, hogy miután ez megvan, azután szoktunk A => C dedukciók alapján még más dolgokat is következtetni, azaz a tudásunkat használjuk más dolgok kikövetkeztetésére is, illetve, amennyiben c-t meg lehet erősíteni, az egy független igazolása istennek (istennél ez is hiányzik).

ostoba vagy, hiszen ez a nomologikus-deduktív modell, és te még ezt se fogod fel, pedig jól dokumentált.

brandon1 2017.06.30. 22:35:54

@Brendel Mátyás: "természettörvények nem léteznek, hanem érvényesek" Már ez is egy hülyeség. Ha elfogadod az igazság korrespondenciaelméletét, akkor el kell fogadnod azt is, hogy a természettörvényeket leíró mondatokat a világban létező tények teszik igazzá, mert ezeknek felelnek meg.

"miért létezik isten?" Itt nincs szükség isten létezésének indoklására, a kérdés az, hogy a tapasztalatunk mitől rendszeres és konzisztens. A teista szerint a válasz az isteni akarat következetességében kereshető.

"mi igazolja, hogy isten a természettörvények magyarázata, és nem más? ugyanis erre sincs független igazolásod." De van, mert ez a következtetés a legjobb magyarázatra típusú érv. Sokkal jobb magyarázat ugyanis a tapasztalat eredetére és rendszerességére, mint a megismerhetetlen anyagi szubsztancia.

"ez olyan hülye indoklás, mintha azzal jönnél"

Egyáltalán nem hülye indoklás. A tudományban elfogadott az idealizáció módszere. en.wikipedia.org/wiki/Idealization

Hülye indoklás az, amit te adtál: "Mi elsőre azt látjuk, hogy a tapasztalataink ilyenek, ezalapján feltételezzük, hogy a realitás ilyen. Ez elég kézenfekvő" Kézenfekvő a faszt. Már Hume leírta, hogy a testek folyamatos és független létezésében az emberek nem filozófiai érvek alapján, hanem ösztönösen hisznek. Ez nem filozófia, mert ebben Hume szerint a gyerekek meg a parasztok hisznek a legösztönösebb módon. És úgy látszik, te is.

szemet 2017.07.01. 06:53:30

@brandon1: "Hume szerint a gyerekek meg a parasztok hisznek a legösztönöseb módon"

Lényegi vita fonalán nem sok jelentősége van ennek de:

Pont sok gyerekeknél van egy korszak amikor ezt erősen teszt alá veszik: pl. hogyha behunyják a szemüket eltűnnek-e ők, a dolgok, a feléjük repülő labda stb... és utána vonnak le következtetést. Szóval szerintem van ösztönös Berkeley korszak is. ;)

A másikra sajnos nem sikerült előásni de rémlik egy jó idézet (nagy matematikus Euler vagy Gauss mondta talán - asszem mindkettő idealizmus kritikus volt), és valahogy úgy szólt, hogy ha egy filozófus azt mondja a fára, hogy nem létezik mindenki vállára emeli és ünnepli - milyen elmés, de ha egy paraszt mondja ugyanezt az uraság hozzákötözi ahhoz a fához és jól megostorozza...

Brendel Mátyás · http://ateistaklub.blog.hu/ 2017.07.01. 07:41:37

@brandon1: "Ha elfogadod az igazság korrespondenciaelméletét, akkor el kell fogadnod azt is, hogy a természettörvényeket leíró mondatokat a világban létező tények teszik igazzá, mert ezeknek felelnek meg."

1) a világban bizonyos dolgok léteznek. tárgyak, és más entitások. elemi részecskék (hullámcsomagok), az elemi részecskékből felépülő atomok, az atomokból felépülő tárgyak, stb.

2) ezeknek a tárgyaknak vannak tulajdonságaik, például tömegük, vagy a téridőben változó pozíciójuk. ezek a tulajdonságok az entitások tulajdonságai, tehát nem függetlenül létező dolgok.

3) a tények ilyen tulajdonságok együttesét leíró mondatok. tehát nem, a tények sem függetlenül létező dolgok, a tények az entitások tulajdonságait leíró igaz mondatok. a mondatok nem létező dolgok. a mondatok az ember "kreálmányai", de a mondatok sem léteznek olyan értelemben, mint a tárgyak. a mondatok igazak vagy hamisak.

4) a természettörvények a tények összefüggéseit leíró mondatok, amelyek igazak. tehát megint nem külön létező dolgok. a természettörvények szintén nem olyan külön, és embertől függetlenül létezők, amiket létre kellett valakinek hoznia valakinek az emberektől függetlenül. a természettörvényeket, mint mondatokat, amelyek helyesen leírják a tények összefüggéseit, az ember "hozta létre".

"Itt nincs szükség isten létezésének indoklására"

ja, bazdmeg, nem egyenlő feltételekkel versenyzünk? az egyébként szigorúan véve nem is létező természettörvényeket indokolni kell, de a hívők szerint kurvára közönséges entitásként, sőt, személyként létező isten létezése az simán maradhat indokolatlanul?! úgy akarod összehasonlítani a két "elméletet", hogy eleve lejt a pálya?! ki vagy te, Orbán Viktor, bazdmeg?!

"A tudományban elfogadott az idealizáció módszere. en.wikipedia.org/wiki/Idealization"

mint egy modell, amelynek egy jelenség közelítően megfelel, de az már nem, hogy az idealizált modell alapján képzelegjél más létezőkről analógiásan.

" Már Hume leírta, hogy a testek folyamatos és független létezésében az emberek nem filozófiai érvek alapján, hanem ösztönösen hisznek."

Hume ezekben a dolgokban tévedett, egy extrém szkeptikus felfogása volt. a tudomány nem extrém módon szkeptikus.

az extrém szkepticizmus oda vezet, hogy nem lehet tudásod semmiről.
a normál szekpticizmus a tudományhoz vezet, ahol van konzisztens és konszenzuális tudásod
az extrém naivitás a hithez vezet, ahol egymásnak ellentmondó hülyeségekben hisznek a vallások.

ateistaklub.blog.hu/2013/09/08/vajon_hume_ateista_volt

nem sikerült válaszolnod a sok kérdésre, amire az isten-"elméletnek" válaszolnia kéne, de nem tud.

1) mi a kurva anyád a célja istennek a világgal?
2) hogy teremt isten Univerzumot, és természettörvényeket? hogy működteti azokat? mi garantálja a természettörvények igazságát, ha istennek van erről akarata? hogy lesz az akaratból törvény?
3)mi isten személyiségének hardvere?
4) honnan ered isten (hardvere)?
5) mik az isten személyiségét, és a hardverének viselkedését leíró törvények? vagy egyszerűbben: mi határozza meg, és mi kódolja isten akaratát?
6) honnan erednek ezek a természettörvények? mi határozza meg ezeket a természettörvényeket?

brandon1 2017.07.01. 08:59:20

@Brendel Mátyás: "a világban bizonyos dolgok léteznek. tárgyak, és más entitások. elemi részecskék (hullámcsomagok), az elemi részecskékből felépülő atomok, az atomokból felépülő tárgyak, stb."

Ezzel azt írtad le, hogy miben szeretsz hinni. A modern fizika ennek ellentmond, mert eszerint nem létezik objektív és embertől független valóság: quantum systems don’t seem to be definite objects localized in space until we come along to observe them. Experiment after experiment has shown—defying common sense—that if we assume that the particles that make up ordinary objects have an objective, observer-independent existence, we get the wrong answers. The central lesson of quantum physics is clear: There are no public objects sitting out there in some preexisting space. As the physicist John Wheeler put it, “Useful as it is under ordinary circumstances to say that the world exists ‘out there’ independent of us, that view can no longer be upheld.”
www.theatlantic.com/science/archive/2016/04/the-illusion-of-reality/479559/

Brendel Mátyás · http://ateistaklub.blog.hu/ 2017.07.01. 09:25:39

@brandon1: odaírtam, hogy "hullámcsomagok", de jössz a terelő, és igaztalan hülyeségeddel, csak, mert az érdemi vitában nincs érved.

szánalmas vagy. azért hiszel a baromságodban, mert hülye vagy.

szemet 2017.07.01. 09:34:56

@brandon1: Wheeler erősen kisebbségi értelmezést vall. Itt válogathatsz a teljesen működőképes kvantumértelmezések közt a táblázatban ahol observer role=none...
en.wikipedia.org/wiki/Interpretations_of_quantum_mechanics#Comparison_of_interpretations

Szerintem amúgy azok egyszerűbbek is, mert nem kell a: mi számít mérésnek/megfigyelésnek, miért okoz az bármit is - köröket futni.

szemet 2017.07.01. 09:37:44

@brandon1: Egyébként ezósok a Neumann Wignert szeretik ott egyenesen tudat kell a hullámfüggvény összeomláshoz ;) Remélem legalább szereztem neked egy új kedvencet.

gw1456 2023.06.24. 20:26:40

@Brendel Mátyás: Sikerült már lenyelned, hogy a 2022-es fizikai Nobel-díjat olyan eredményért kapta három fizikus, ami a szubjektív idealizmus igazságát támasztja alá? (Lásd a Does color exist when no one is watching? részt az alább belinkelt tájékoztatóban). Őgyhogy az egész szánalmas blogbejegyzésed a te saját nyomorúságodat támasztja alá, nem a szubjektív idealizmusét.

www.nobelprize.org/prizes/physics/2022/popular-information/

Brendel Mátyás · http://ateistaklub.blog.hu/ 2023.06.25. 19:17:08

@gw1456: jajj, már megint milyen idióta vagy!

1) Az Aspect kísérleteket régen végezték el, én nem most értesültem róla, te láthatóan csak most, a Nobel-díj kiosztáskor.

2) "a szubjektív idealizmus igazságát támasztja alá" Semmi ilyesmiről szó sincs, és nem is szerepel az általad megosztott linken.

Nyomorúságos kamu- és hülyegyerek vagy.

Brendel Mátyás · http://ateistaklub.blog.hu/ 2023.06.25. 19:27:02

@gw1456: de fengtebb brandon néven ezt a kört már lefutottad, és pofával estél bele a tehénszarba. úgyhogy felesleges újabb kört futnod.

gw1456 2023.07.12. 18:10:24

@Brendel Mátyás "a világban bizonyos dolgok léteznek. tárgyak, és más entitások. elemi részecskék (hullámcsomagok), az elemi részecskékből felépülő atomok, az atomokból felépülő tárgyak, stb.

2) ezeknek a tárgyaknak vannak tulajdonságaik, például tömegük, vagy a téridőben változó pozíciójuk."

Hülyeség. A téridő nem a térben létezik, hanem matematikai konstrukció. Ebből a tényből az idealizmus melletti érvet lehet összeállítani.Az úgynevezett fizikai tárgyak nem léteznek az elmétől függetlenül. Bármi, amit megtudunk róluk, szintén matematikai természetű.
Ezt már megvitatták itt a fizikusok:
physics.stackexchange.com/questions/454068/is-spacetime-wholly-a-mathematical-construct-and-not-a-real-thing

Brendel Mátyás · http://ateistaklub.blog.hu/ 2023.07.13. 23:02:15

@gw1456: " A téridő nem a térben létezik, "

Még véletlenül sem állítottam olyan hülyeséget, hogy "a téridő a térben létezik".

Fordulj kezelő orvosodhoz!

Brendel Mátyás · http://ateistaklub.blog.hu/ 2023.07.13. 23:44:36

@gw1456: "Az úgynevezett fizikai tárgyak nem léteznek az elmétől függetlenül."

Mit jelent ez a mondat egészen pontosan? Hogyan tudod ezt kísérletileg tesztelni? A kísérlet melyik kimenetele igazolja, és melyik cáfolná ezt az állítást? Ez az állítás ugye a filozófia egyik leginkább rágott csontjai közé tartozik, és azok közé az állítások közé, amelyik erősen metafizikai, és kérdéses az értelmezése, illetve a logikai pozitivisták az ilyen állítások jelentését is kétségbe vonták. És tényleg kérdéses, hogy mit is akar ez jelenteni. Carnap úgy fogalmazott, hogy ez az állítás erősen metafizikai, nagyon nehéz úgy értelmezni, hogy értelmes, fizikai módszerekkel eldönthető kérdés legyen. Az Aufbau végén azt írja le, hogy a fizikalista nyelv az ember egy természetes, jól működő nyelve, ezzel jól le lehet írni a világot. Más nyelveken le lehet írni valami olyasmit, ami ezzel empirikusan ekvivalens, de elég nehéz ilyen nyelveken beszélni, gondolkodni, és aztán kérdéses az is, hogy tényleg teljesen ekvivalens-e.

Van egy olyan igencsak általános, alapvető megfigyelésünk a világról, legalábbis makro-szinten, hogy a megigyeléseinket jól le tudjuk írni azzal, hogy rajtunk kívül létező tárgyak vannak, és ezeknek bizonyos tulajdonságaik vannak, és bizonyos törvények szerint ezek a tulajdonságok konzisztensen megmaradnak, vagy ha változnak, akkor is törvényszerűen. Ezt elég nyakatekerten, valószerűtlenül és nyelvileg elég nehezen lehetne kiváltani valami mással.

Mikro-szinten, a kvantummechanikában pedig ez nem teljesen cáfolódik, de ott valami furcsa dolgok vannak, és pont a micro-szinten merül fel az, hogy az elméleteink csak matematikai konsturkciók, és már kezd elveszni az, hogy mit is jelentenek. Például a húrelmélet, ami évtizedek óta igazolhatatlan és cáfolhatatlan. Meg, hogy brit tudósok minden héten előjönnek valamivel, ami akár lehet igaz is, de igazolni már nem tudják.

" Bármi, amit megtudunk róluk, szintén matematikai természetű."

Ez meg egy még kevésbé megfogható mondat. Mi az, hogy "matematikai természetű"? Ha a tárgyak valóban léteznek, akkor azt hogyan cáfolná az, hogy amit tudunk róluk, azt ma már jól le tudjuk írni matematikailag? Mit mond ez róluk? Semmit. A matematikai egy eszköz a dolgok leírására, és ma már igen fejlett eszköz. Az, hogy metametikával sok mindent le tudsz írni, az nem mond többet, mint az,hogy a mobiltelefonoddal nagyjából mindent le tudsz fényképezni. z csak azt jelenti,hogy jó a mobilod. A világ, az emberek már akkor is voltak, amikor még nem volt matematika, és matematikával akkor még semmit nem lehetett leírni, és az emberek már akkor is azt gondolták, léteznek a tárgyak körülöttünk.

Nagyon hülye vagy. Az első mondatod értelmetlen volt, elbasztad, a második metafizika, a harmadik meg nagyon homály.
süti beállítások módosítása